Сохранен 515
https://2ch.hk/sci/res/271609.html
24 декабря Архивач восстановлен после серьёзной аварии. К сожалению, значительная часть сохранённых изображений и видео была потеряна. Подробности случившегося. Мы призываем всех неравнодушных помочь нам с восстановлением утраченного контента!

Математика, тред №19

 Аноним 31/05/15 Вск 22:00:36 #1 №271609 
14330988366870.jpg
Многие спрашивают, как стать настоящим математиком, который занимается интересными концептуальными вещами. Очень просто:
1. Берешь какой-то список попонтовее. Не дай бог прочитать книгу, которой там нет, рисково это, можно зашквариться.
2. Заучиваешь наизусть определения. Не пытайся обдумывать прочитанное, это занимает слишком много времени, а тебе нужно успеть дойти до настоящей современной математики.
3. Раз в неделю заходи на архив, выбирай рандомную статью и несколько секунд читай несколько первых строк. Особый шик — запомнить название. Так ты приобщаешься к современным исследованиям. На забудь скачать статьи Мочидзуки.
4. Ежедневно посещай блог вербита и математические треды. Глубокие обсуждения математических проблем в среде настоящих специалистов помогут повысить уровень математической культуры для недосягаемых академическому быдлу высот.
5. Настоящая математика только на английском. Постепенно прекращай читать книги, написанные по-русски, да и вобще любые, зато показывай всем, как ты знаешь английский, а не то примут за картофана.
6. Интегралы, которые тебе задают на первом курсе, решать не обязательно, это называется вторая культура.
7. Ну и что, что тебя выперли из вузика? Это и вузом то назвать было нельзя, так, картофельная плантация, работники которой реконструируют математику допотопных времен. Вон, вербита тоже выперли. Настоящее образование только за границей. Злая судьба забросила тебя в водочный ад, но рано или поздно, благодаря своему упорному труду ты выберешься в Гарвард, где будешь раз в неделю ходить на семинары по теарам Ходжа по выбору.
8. К этому моменту можно самому составлять списки, давать советы, отвечать на вопросы. Списки составляются очень легко: берешь готовый от вербита и меняшь местами названия. Можно взять парочку и перемешать.
9. Отвечать на вопросы тоже очень просто: если увидел знакомые слова — перепечатывай соответствующие определения из книги. Если незнакомые — это картофан, гони его ссаными тряпками.
В точности следуя этим советам ты станешь Настоящим Первокультурым Математиком и сможешь посвящать свободное от жалоб мамки время на улучшение научной атмосферы в парашке путем остроумных дискуссий в сети.

Прошлый: >>268391
Аноним 31/05/15 Вск 22:06:10 #2 №271612 
>>271609
Проиграл с оп-пика
bydlo 31/05/15 Вск 22:07:54 #3 №271613 
>>271609
Очень крутая паста, абсолютно серьезно. Попробую вбросить какое-то подобие "задач". Точнее даже не задачи, а вопросы и тонкие моменты, с которыми я очень долгое время тупил, постараюсь без баянов. Для начала.
1. Пусть f(t),g(t) - гладкие функции R->R, значит ли это, что, кривая, заданная параметрически как t -> (f(t),g(t)) - это гладкая кривая в R^2?
Аноним 31/05/15 Вск 22:08:22 #4 №271614 
>>271609
Вопрос тупой наверное, но мне не у кого спросить:
Допустим надо раскидать по кластерам точки N-мерного пространства, если я используя разные метрики (евклид, канберра, минковского ... ) получаю в точности одно и тоже распределение по кластерам для нескольких различных алгоритмов кластеризации (минимальная дисперсия Варда, метод ближнего соседа, метод полной связи, UPGMA, WPGMA, WPGMC, UPGMC) - то это возможная ситуация, или я где-то серьезно обосрался?
Ну то-есть для двух разных методов кластеризации результат естественно разный, но вот если берем один метод и гоняем разные метрики - получаем одинаковое распределение точек по кластерам - это нормальная ситуация?
Аноним 31/05/15 Вск 22:21:48 #5 №271618 
>>271613
Ну да, по определению же.
Частные производные координат непрерывны.
Знаю, что такое схема, но не сходу ответил лол.
bydlo 31/05/15 Вск 22:23:08 #6 №271619 
>>271618
Какое у тебя определение гладкой кривой? Я понимал всегда, гладкая кривая = гладкое 2-многообразие
bydlo 31/05/15 Вск 22:26:53 #7 №271620 
>>271619
>гладкое 2-многообразие
гладкое 1-многообразие, fix
Аноним 31/05/15 Вск 22:35:12 #8 №271621 
>>271613>>271618
Конечно, нет. Пример (t^2, t^3)
Буду теперь подписываться, а то на все вопросы отвечаю страждущих, а потом картофан выебывается.
Вербиторебенок
bydlo 31/05/15 Вск 22:36:58 #9 №271622 
>>271621
Овкос!
2. Какое условие (как можно более слабое) нужно наложить на f,g, чтобы t -> (f(t),g(t)) всегда было гладкой кривой?
Аноним 31/05/15 Вск 22:52:32 #10 №271624 
>>271622
Дифференциал в ноль не обращается, точн.
Чет я совсем
Аноним 31/05/15 Вск 22:52:38 #11 №271625 
Чем мотивировано название "сепарабельный многочлен" Из его определения непонятно, почему выбрано именно слово "сепарабельный".
Аноним 31/05/15 Вск 22:53:10 #12 №271626 
>>271624
Тогда теорему о неявной функции можно юзать.
bydlo 31/05/15 Вск 23:16:05 #13 №271627 
>>271624
>>271626
Accept!
3. Пусть у нас есть гладкие (!) многообразия A и B и они гомеоморфны, следует ли из этого, что они диффеоморфны?
Аноним 31/05/15 Вск 23:21:20 #14 №271629 
>>271627
https://en.wikipedia.org/wiki/Exotic_sphere
bydlo 31/05/15 Вск 23:34:03 #15 №271633 
>>271627
>>271626
>>271624
Кстати я чуть поспешил. Чтобы использовать теорему о неявной фунции нужно всё-таки требовать не тождественность дифференциала нулю, а кое что чуть посильнее.
>>271629
Вроде того. 3:


Аноним 31/05/15 Вск 23:36:10 #16 №271634 
Он довел, я бы сказал, до логического предела подход Арнольда, согласно которому всякую задачу нужно формулировать в том минимальном варианте, в котором она сохраняет принципиальный интерес, но еще не решена. Арнольд отстаивал этот тезис в противовес французской школе, школе Бурбаки, в которой принято формулировать задачи в максимально возможной общности.
bydlo 31/05/15 Вск 23:37:32 #17 №271636 
4. Существует ли поверхность (2-многообразие без края, особенностей, связное) в R^3 такая, что какой-то из её гомеоморфизмов на себя не может быть никак продолжен до гомеоморфизма всего R^3 на себя?
bydlo 31/05/15 Вск 23:41:42 #18 №271638 
>>271633
>не тождественность
не нетождественность
Аноним 01/06/15 Пнд 00:08:19 #19 №271643 
>>271625
Сепарабельным расширением.
Аноним 01/06/15 Пнд 01:22:39 #20 №271648 
>>271621
>(t^2, t^3)
И что в ней не гладкого? Производная обращается в ноль — похуй, главное дифференцируема. Условие необращения производной в 0 вроде регулярностью называется.
Аноним 01/06/15 Пнд 01:34:09 #21 №271649 
>>271636
Вроде перестановка координат в S^1 x S^1 у тора не продолжится. Внутренняя часть тора (S^1 на диск) перейдёт во внутреннюю, а тогда вторая координата по-прежнему перейдет в нечто стягиваемое по внутренней части. Так что в фундаметальной группе Z^2 бывают только диагональные гомоморфизмы.
Вербитодаун
Аноним 01/06/15 Пнд 01:35:39 #22 №271650 
>>271648
Какое у тебя определение гладкой кривой? Нарисуй эту хуйню, у тебя будет в нуле стремный угол.
Аноним 01/06/15 Пнд 01:46:37 #23 №271653 
>>271625
Да и так понятно, ведь separate - разделять, типа корни разделены (т.е кратных корней нет).
bydlo 01/06/15 Пнд 02:20:28 #24 №271655 
>>271649
Существует ли замкнутая кривая в R^3 такая, что какой-то из её гомеоморфизмов на себя не может быть никак продолжен до гомеоморфизма всего R^3 на себя?
bydlo 01/06/15 Пнд 02:20:50 #25 №271656 
>>271649
И да, пример правильно, конечно же.
Аноним 01/06/15 Пнд 03:24:18 #26 №271665 
>>271655
Вроде да. Замкнутая кривая диффеоморфна S^1. Можно взять трубчатую окрестность, которая будет тривиальным расслоением над S^1, так как ориентируемо, а над окружностью их всего два, второе неориентируемо. А тогда на нее продолжение очевидно (x,y)->(f(x),y), так как это просто S^1 на диск. А дальше надо плавно перейти на тождественное в стиле разбиения единицы так, что на границе оно будет тождественным. Тогда далее продолжим уже id.
Аноним 01/06/15 Пнд 04:55:57 #27 №271666 
>>271665
>Вроде да
В смысле всегда продолжается. Не увидел вопрос.
Аноним 01/06/15 Пнд 07:29:06 #28 №271668 
14331329464000.jpg
>>271650
Аноним 01/06/15 Пнд 08:12:36 #29 №271670 
>>271668
Гладкая кривая и гладкая параметризация разные вещи все-таки.
Аноним 01/06/15 Пнд 09:05:25 #30 №271678 
Двач, в одной из предыдущих тем анон выкладывал схему разделов современной математики то ли списком то ли в виде дерева, не помню. Выложи, пожалуйста, еще раз, если у тебя сохранилась.
Аноним 01/06/15 Пнд 10:49:23 #31 №271691 
Как всё-таки латеху совсем запретить растягивать пробелы?
Аноним 01/06/15 Пнд 12:17:38 #32 №271694 
14331502580680.jpg
>>271678 Самому лень пролистать пару тредов по шапкам?
Аноним 01/06/15 Пнд 13:03:30 #33 №271699 
>>271694
Че за инвалид это писал?
Аноним 01/06/15 Пнд 13:14:50 #34 №271700 
Ребята, а насколько глубоко нужно для начала знать общую и алгебраическую топологию, если хочешь пойти по алгебраической ветке? Поподробней если можно, до куда можно дойти, а дальше уже дропнуть?
Аноним 01/06/15 Пнд 14:08:00 #35 №271703 
>>271699
Весьма здравая картинка на самом деле. Ясно, что на текущий момент нереально быть хорошо знакомым со всей современной математикой, но название и не претендует на полноту. Никакого явного бреда в ней не видно. Например, как логик могу заявить, что его деление логики несколько архаично и я бы делил ее не так, но не то, чтобы такое деление полностью не соответствовало реальности.
Аноним 01/06/15 Пнд 14:27:19 #36 №271706 
>>271703
Автор картинки говорил, что это не деление, а просто некоторые подразделы.
Аноним 01/06/15 Пнд 15:05:12 #37 №271710 
>>271703
>его деление
Это же классическое деление.
bydlo 01/06/15 Пнд 16:30:39 #38 №271732 
>>271665
Не очень уверен, но у меня есть контрпример кажись.
Аноним 01/06/15 Пнд 16:53:24 #39 №271737 
14331668048640.png
>>271609
Зачем хуй с оп-пика сидит на тянке?
Аноним 01/06/15 Пнд 19:18:48 #40 №271763 
>>271700
Читай себе хатчера на досуге и все, я думаю этого хватит вполне. Ну а как не хватит, то ты сам уже поймешь.
Аноним 01/06/15 Пнд 19:23:37 #41 №271764 
>>271737
Потому что она мягкая:)
Аноним 01/06/15 Пнд 19:34:16 #42 №271772 
Анон, что делать, если у меня такая проблема: разберу конец доказательство - забываю начало; вернусь к началу - забываю конец? Как эта хуйня лечится?
Аноним 01/06/15 Пнд 19:36:40 #43 №271774 DELETED
>>271772
Нет причин помнить весь текст, если тезис каждого предложения вытекает из предыдущего.
Аноним 01/06/15 Пнд 19:40:18 #44 №271775 
>>271774
Значит, я не могу выводит следующий тезис из текущего. А эта хуйня как лечится? Иногда в доказательствах такие повороты, что только удивляешься, как математик до этого догадался, и хочется понять сначала ход мыслей математика, как он дошел до этого приема, потому что иначе остается только зубрить.
Аноним 01/06/15 Пнд 19:46:50 #45 №271776 DELETED
>>271775
А это лечится только опытом. Читай мат-литературу, расширяй количество доступных тебе абстракций и паттернов, решай задачи (желательно практические, чтобы решение тебе действительно было нужным, чтобы это была внутренняя, а не внешняя потребность).
Аноним 01/06/15 Пнд 19:54:22 #46 №271778 
>>271775


Почитай Пойа "Математика и правдоподобные рассуждения ". Может, станет маленько яснее.Но если большинство доказательств вызывают большие трудности, то не исключено, что тебе лучше заняться не математикой, а чем-нибудь другим - чего мозги насиловать?
Аноним 01/06/15 Пнд 20:03:30 #47 №271780 
Приведите пример не квази-проективного алгебраического многообразия
Аноним 01/06/15 Пнд 20:05:31 #48 №271782 
>>271778
>чего мозги насиловат
Потому что мне нравится математика.
Аноним 01/06/15 Пнд 20:12:48 #49 №271783 
>>271782
Ой зря ты это
Я в свое время тоже так начинал, теперь вот жалею. Почти все время уходит на математику а, перспектив-то нет. И пути назад нет.
Аноним 01/06/15 Пнд 20:12:53 #50 №271784 
>>271782
Тогда читай Пойа. Еще помогает пытаться придумывать доказательства самостоятельно и вообще идти не тем путем, как в учебнике. Либо посмотреть несколько учебников и понять, ккой стиль изложения тебе больше нравится.Разные люди соображают по-разному на довольно базовом уровне - запросто может быть, что стиль автора учебника тебе не подходит.
Аноним 01/06/15 Пнд 20:17:26 #51 №271787 
>>271783
>Я в свое время тоже так начинал, теперь вот жалею. Почти все время уходит на математику а, перспектив-то нет
А мне больше и некуда тратить свое время. Разве что на книжки или аниме. Хотя мб если бы был более инициативным год назад, тратил бы время еще и на тянку, она сама со мной тогда гулять пошла.
Аноним 01/06/15 Пнд 20:19:30 #52 №271788 
Суп, Саймат. Где почитать про равносильные преобразования систем нелинейных уравнений?
Аноним 01/06/15 Пнд 20:27:00 #53 №271791 
>>271788
Я думаю тебе стоит уточнить тип систем таки.
Аноним 01/06/15 Пнд 20:29:52 #54 №271792 
>>271791
Эмм... А какие есть? Гугл молчит.
Аноним 01/06/15 Пнд 20:31:07 #55 №271793 
>>271792
А, нет, что-то есть
Аноним 01/06/15 Пнд 20:31:29 #56 №271795 
>>271792
Алгебраических
Дифферинциальных(тысячи подтипов)
Диофантовых
.......
Аноним 01/06/15 Пнд 20:35:42 #57 №271797 
>>271795
Ну и эквивалентность можно по разному понимать
Аноним 01/06/15 Пнд 20:42:54 #58 №271798 
>>271795
Диофантовы
Алгебраические изи же, приравнял да и всё
Аноним 01/06/15 Пнд 20:54:34 #59 №271801 
>>271797
А под эквивалентностью имеется ввиду эквивалентность систем
Аноним 01/06/15 Пнд 20:54:56 #60 №271802 
>>271783
> перспектив-то нет
Почему?
>И пути назад нет.
Иди в программисты.
Аноним 01/06/15 Пнд 20:59:10 #61 №271803 
>>271798
хуя ты гротендик
Аноним 01/06/15 Пнд 21:08:17 #62 №271805 
>>271803
Ну если имеются ввиду системы вида {f(x)=0 и g(x)=0 и h(x)=0} то изи же, разве нет? Или я не так понял? В любом случае, не бейте школьника, расскажите лучше про нелинейные диофантовы.
Аноним 01/06/15 Пнд 21:15:40 #63 №271806 
>>271805
Ну понимать свойста решений - не изи.
Решать - не изи.
Ну диофантовы это же те же алгебраические, над Z только.
Возможно имеет смысл смотреть в сторону базисов Гребнера. Не знаю.

Аноним 01/06/15 Пнд 21:28:27 #64 №271809 
>>271806
Ой ой, че это я залупил. Не диофантовы, алгебраические. Но с двумя и более переменными.
Аноним 01/06/15 Пнд 21:30:58 #65 №271810 
>>271809
И без матана
Аноним 01/06/15 Пнд 21:41:38 #66 №271813 
>>271732
Я тоже в своем рассуждении не очень уверен, там многое надо расписать подробнее. Так что покажи контрпример.
Аноним 01/06/15 Пнд 22:25:22 #67 №271818 
Кто-нибудь может подсказать годное руководство по картофельному матану. Уровень первого семестра. Осталось просто несколько дней до экзамена и надо научиться все считать.

мимопрограммист

Аноним 01/06/15 Пнд 22:31:51 #68 №271819 
>>271818
>Осталось просто несколько дней до экзамена и надо научиться все считать
Ты соснул.
Аноним 01/06/15 Пнд 22:32:15 #69 №271820 
14331871358230.jpg
>>271787
>она сама со мной тогда гулять пошла
Хуясе, альфач в треде.
Аноним 01/06/15 Пнд 22:55:11 #70 №271823 
>>271820
Просто я няшный, а она еще и длинноволосых любит. Но скорее всего она бы все равно ушла из-за моего синдрома Аспергера.
Аноним 01/06/15 Пнд 22:57:26 #71 №271824 
>>271823
>Аспергер
А, так вот почему ты аутистично долбишь математику.
Аноним 01/06/15 Пнд 23:27:43 #72 №271827 
>>271823
Так вот что со мной!!
Аноним 01/06/15 Пнд 23:33:04 #73 №271828 
Итак, мы выяснили, что математик уровня /б - это задрот и девственник.
Аноним 01/06/15 Пнд 23:39:09 #74 №271829 
>>271828
>математик уровня /б - это задрот и девственник
Ну ты прям Америку открыл.

>>271824
>вот почему ты аутистично долбишь математику
Хуёвый из него долбитель - даже не может проследить цепочку доказательства, а это должно быть интуитивно понятно любому мало-мальски разбирающемуся в математике.
Аноним 01/06/15 Пнд 23:39:19 #75 №271830 
>>271828
Так как бы это аксиома, которая всем должна быть уже давно известна.
Аноним 01/06/15 Пнд 23:54:42 #76 №271832 
>>271829
>даже не может проследить цепочку доказательства
Проследить - могу, воспроизвести по памяти - сложно, особенно если теорема из абстрактной дрочильни типа коммутативной алгебры и начал алгебраической геометрии.
Аноним 01/06/15 Пнд 23:56:56 #77 №271833 
>>271832
Приведи пример того, что вызвало у тебя затруднение?
Аноним 02/06/15 Втр 00:18:00 #78 №271834 
>>271828
я не девственник например
Аноним 02/06/15 Втр 00:49:03 #79 №271836 
>>271834
Ну и чему ты радуешься? Наоборот, плакать надо. Теперь ты не чист, ты не сможешь отдать всю свою любовь науке.
Аноним 02/06/15 Втр 00:53:09 #80 №271837 
>>271836
Не то чтоб я прям радовался, просто опровергаю утв. анона.
Ну и это только помогает. Типо когда ты не девственник, то ты хочешь это исправить. А тут уже попробовал и все, можно 'науке отдаваться'
Аноним 02/06/15 Втр 00:53:56 #81 №271838 
>>когда ты девственник
fix
Аноним 02/06/15 Втр 00:55:23 #82 №271840 
14331957237630.jpg
>>271834

Аноним 02/06/15 Втр 00:59:22 #83 №271842 
>>271837
>можно 'науке отдаваться'
Это ты про ЕГЭ?
Аноним 02/06/15 Втр 13:08:30 #84 №271905 
>>271613
Чего-то я не понимаю... Гладкость это же касательные, производные и вся такая хуйня, то есть вторая культура. Зачем такие задачи решать?
Аноним 02/06/15 Втр 13:35:39 #85 №271908 
>>271621
(x,y) = (t^2; t^3)
y= t3 = t2*3/2 = x3/2

И что, не гладкое?
Аноним 02/06/15 Втр 14:02:42 #86 №271914 
>>271908
Ну так найди производную в нуле-то
Аноним 02/06/15 Втр 17:02:51 #87 №271936 
14332537713730.jpg
>>271833
В начале года надо было разобрать пикрелейтед. Долго думал над выделенным местом, потому что у меня привычка разбирать теоремы пошагово, а не перескакивать через непонятные места. И если что-то непонятно почему это мы вдруг решили возвести в степень n'? какое озарение было у математика, что он решил сделать именно так?, меня раздражает, что это не сопровождается комментариями, и приходится читать до самого конца с давящим чувством, что позади не все понятно.
Аноним 02/06/15 Втр 17:27:58 #88 №271937 
>>271936
>Долго думал над выделенным местом, потому что у меня привычка разбирать теоремы пошагово, а не перескакивать через непонятные места.
Откуда такая привычка пошла? Думаю, это несовсем верно, даже вредно: у тебя складывается иллюзия, что ты действительно что-то понял и не нуждаешься в переосмысливании прошлого опыта.
Аноним 02/06/15 Втр 17:34:39 #89 №271939 
>>271937
Михаил Сергеевич, залогиньтесь.
Аноним 02/06/15 Втр 17:35:42 #90 №271940 
>>271936
Дак это вообще интуитивно очень понятно. Циклическая группа порядка n – это степени комплексного числа s=exp(i*2pi/n) – n точек на единичном круге, с операцией ( комплексного) умножения. Пусть n на что-то делится (скажем, n четное ). Тогда степени для некоторых точек на окружности не будут описывать полную группу (нарисуй для n=6, скажем – для s^2 точек будет 3 а не 6, а s³ =-1 , и степени будут прыгать в плюс-минус единицу) . Понятно, что все это связано с делителями n, отсюда формула почти очевидна – дальше ее надо только доказать.
Аноним 02/06/15 Втр 18:47:18 #91 №271945 
>>271614
Да, нормальная ситуация, способы кластеризации отличаются друг от друга скоростью схождения, уязвимостью к локальным экстремумам и т.д., но в пределе реализуют один и тот же регрессионный анализ.
Аноним 02/06/15 Втр 19:59:30 #92 №271947 
>>271937
>Откуда такая привычка пошла?
Потому что мне нравится ощущение ясности в голове, когда теорема уложилась в памяти и находится в виде ясного образа, к любому месту которого можно обратиться и четко вспомнить доказательство в этом месте. Я даже для этого заведения слишком упорот?
Аноним 02/06/15 Втр 20:12:43 #93 №271949 
14332651632530.jpg
>>271939
Кстати, да, я услышал что-то такое у него. Можешь скинуть эту статью?

>>271940
>Дак это вообще интуитивно очень понятно.
Только для опытного математика.

>>271947
Кто как хочет.
Любишь много внимания к твоей персоне, извини.
Аноним 02/06/15 Втр 20:14:15 #94 №271950 
>>271947
Слишком перфекционист.
Аноним 02/06/15 Втр 20:16:17 #95 №271951 
>>271949
Кстати говоря слышал подобные советы не только от Миши.
Еще один математик при мне говорил, что полезно окунаться в дебри науки, даже если не все понятно. И не обязательно во всем сразу досконально разбираться, со временем появиться необходимая интуиция и понимание.
Бтв я так и делал. И вроде и впрямь ничего.
Если через какое-то продолжительное время поймешь, что у тебя остались таки пробелы, то вернись и прочитай еще раз. Будет уже гораздо проще и скорее всего все поймешь.
Это не значит, что надо читать математику, как дешевый роман, но все же не сидеть вечерами над одной страничкой.
Аноним 02/06/15 Втр 20:20:46 #96 №271953 
>>271949
>Вторая проблема, тоже весьма неприятная люди любят читать кни-
ги подряд. При этом дойдя до места, где непонятно, люди читают это ме-
сто снова и снова, до полного отупения. Это неправильно! Надо открыть
книжку на другом месте и читать там, а непонятное место перечитать
потом.
Нашел в его "Топологии".
>>271951
Согласен, так интересней. Компромисс между движением вперед и ясностью достигается желанием стать скорее крутым спецом.
Аноним 02/06/15 Втр 21:49:28 #97 №271963 
>>271947
>Я даже для этого заведения слишком упорот?
Нормально всё с тобой. Ты на вербитодетей попал просто, а им от Мишиных канонов отходить не позволено.
Аноним 03/06/15 Срд 00:10:24 #98 №271978 
14332794248850.jpg
Я мимокрокодил, поэтому мне ваши срачи по поводу культур в математике, картофана и прочего не интересны. Я к вам с реквестом годной вводной книги по теории категорий(на инглише). Спасибо заранее и не ссорьтесь.
Аноним 03/06/15 Срд 00:24:28 #99 №271980 
>>271978
Маклейн, который придумал теорию категорий, написал по ней годный учебник. Что тебе ещё нужно?
Аноним 03/06/15 Срд 00:34:20 #100 №271984 
>>271978
Если вводной, то лучше Lavwere, Сonceptual Math. Маклейн - больше для продвинутых.
Аноним 03/06/15 Срд 01:20:23 #101 №271991 
В последнее время что-то начинают дохуя пиздеть за нормальных пацанов ебать.... Пиздец, мля когомологи продроченые) Нет что б идти кодить, поднимать нормально денег....отдыхать нормально нет сидят в своих голономиях) голожопые бля! Ну и сидите не пиздите тогда, если у ребят нормальное настроение будет....пронесет....а так вешайтесь.
Аноним 03/06/15 Срд 01:29:46 #102 №271993 
>>271980
То, что сказал анон ниже. Мак Лейн слишком всеобъемлюще написал. Видела отсылки к нему в другом учебнике, тяжко.
>>271984
Спасибо
Аноним 03/06/15 Срд 04:11:29 #103 №271999 
14332938890330.jpg
Ребят, посоветуйте какую-нибудь книгу вроде алгебры Гельфанда с Шенем, но только по геометрии планиметрии. Что-нибудь такое же простое и занимательное, но ёмкое.
Аноним 03/06/15 Срд 05:11:04 #104 №272001 
>>271999
Шень, Геометрия.
Аноним 03/06/15 Срд 09:07:41 #105 №272009 
14333116613520.jpg
>>271999
Где взять ещё её фоточек?
Аноним 03/06/15 Срд 09:41:51 #106 №272015 
>>272009
В /g/.
Аноним 03/06/15 Срд 11:10:56 #107 №272036 
>>271991
слишком толсто имитируешь речь быдла.
bydlo 03/06/15 Срд 11:19:52 #108 №272041 
>>272036
А по мне так очень клёво вышло! (писал не я).
>>271813
Пример не мой, извиняюсь:
http://arxiv.org/abs/0910.4949v2
Аноним 03/06/15 Срд 15:57:39 #109 №272098 
Что означает "множество с определенной структурой"? В чем конкретно выражается эта структура, в каких то соответствиях между элементами?
Аноним 03/06/15 Срд 16:26:54 #110 №272101 
>>272098
Не совсем. Собственно, главное выражение структуры в математике — член Ашота, нежно оплодотворяющий твою мамашу.
Аноним 03/06/15 Срд 16:30:06 #111 №272102 
>>272101
Проиграл с подливой.
Аноним 03/06/15 Срд 17:16:13 #112 №272106 
Матемач, смотри задачу: найти такое n, при котором существует число x, при этом x^1 пренадлежит (1;2), x^2 - (2;3) ... x^n - (n;n+1)
Аноним 03/06/15 Срд 17:54:00 #113 №272112 
14333432405110.jpg
Го соцопрос
1. Вербит или картоха
2. Готика или тес
Аноним 03/06/15 Срд 18:10:13 #114 №272115 
>>272098
Структура векторного пр-ва, структура группы, структура топологического пр-ва, етк
Есть и формальное определение, но кажется уже после примеров все понятно.
Аноним 03/06/15 Срд 18:16:47 #115 №272117 
>>272112
2. Готика жива, смерть недалека.
бывший гот

Я бы согласился потерять часть памяти ради того, чтобы снова ощутить те эмоции от игры в готику
Аноним 03/06/15 Срд 18:30:10 #116 №272119 
>>272112
Нахуй иди, школотун.
Аноним 03/06/15 Срд 19:12:56 #117 №272123 
Анон, до сих пор мне матан нравится не так сильно, как алгебра и производные от нее, но недавно я сам увлекся красотой математической теории поля и векторного анализа. Посоветуй задачник, чтобы хорошо в ней разобраться.
Аноним 03/06/15 Срд 19:15:53 #118 №272125 
>>272106
Любое n от 1 до 4. В последнем случае x можно брать любое от 3^(1/3) до 5^(1/4).
Аноним 03/06/15 Срд 19:21:07 #119 №272126 
>>272123
>красотой математической теории поля и векторного анализа.
Ну и чем они красивее, например, гомотопической алгебры?
https://youtu.be/mqAf5lOJZew
Аноним 03/06/15 Срд 19:29:22 #120 №272129 
14333489620540.jpg
>>272126
Тем, что поля связывают все во вселенной, мне очень доставляет мысль о том, что когда-нибудь я разберусь в теории поля и буду острым и мудрым взором смотреть на вселенную.
Аноним 03/06/15 Срд 19:29:52 #121 №272130 
>>272126
>>272123
Есть два стула, на одном векторные поля точены, на другом гомологии дрочены, на какой сядешь, на какой мать посадишь?
Аноним 03/06/15 Срд 19:31:48 #122 №272131 
>>272119
Псс
>>272117
А по категориям угараешь?
Аноним 03/06/15 Срд 19:34:53 #123 №272133 
14333492934480.webm
>>272130
Правильный ответ.
Аноним 03/06/15 Срд 19:36:34 #124 №272134 
>>272131
>А по категориям угараешь?
Закончится курс алгебры как база для других алгебраических курсов - подумаю, чем угореть. Пока решил летом обмазываться топологией, потому что она всплывает чуть менее чем везде.
Аноним 03/06/15 Срд 19:39:33 #125 №272135 
>>272134
А как ты относишься к событиям на Украине?
Аноним 03/06/15 Срд 19:40:29 #126 №272136 
>>272135
Похуй.
Аноним 03/06/15 Срд 20:20:03 #127 №272143 
Матоны, а за два месяца можно нахвататься верхов чтобы никто не отличил меня от вербитятка?
sageАноним 03/06/15 Срд 20:21:20 #128 №272144 
>>272143
Ты для себя спрашиваешь или для друга? Сколько другу лет, какое образование?
Аноним 03/06/15 Срд 20:31:19 #129 №272148 
>>272144
Для себя
Аноним 03/06/15 Срд 20:31:59 #130 №272149 
>>272143
>два месяца
Двух тредов будет достаточно.
sageАноним 03/06/15 Срд 20:35:55 #131 №272150 
>>272148
Сколько лет, какое образование? Впрочем, да, уже понятно, что тебе двух месяцев не хватит точно.
Аноним 03/06/15 Срд 20:41:19 #132 №272152 
>>272150
Почему?
sageАноним 03/06/15 Срд 20:45:52 #133 №272156 
>>272152
Нет в тебе творческой искорки, нет того полёта мысли, что необходим для постижения математических тайн.
Аноним 03/06/15 Срд 20:47:32 #134 №272157 
>>272156
Нет есть искорка. Образование высшие, михмат.
sageАноним 03/06/15 Срд 20:50:57 #135 №272158 
>>272157
Если есть искорка, то ты уже знаешь ответ, сколько времени потратишь на математику. Всю жизнь.
Аноним 03/06/15 Срд 20:51:44 #136 №272159 
>>272156
Хуево это все. Нет тян - нет и искорки, которая никогда не в тебе не появится, пока ты в плохом настроении. Помню, как я в школе угорал по анализу... такое удовольствие было...
Аноним 03/06/15 Срд 20:53:49 #137 №272160 
>>272159
>тян
С говном чан. Стоило один раз про биопроблемы сказать в треде, так начинается. Так что там с искоркой на два месяца?
Аноним 03/06/15 Срд 20:58:02 #138 №272161 
>>272159
>угорал по анализу
Так это же картофан! Ололо все вербитобляди школозашкварены.
sageАноним 03/06/15 Срд 21:01:36 #139 №272162 
>>272159
Да, удовлетворять потребности из пирамиды Маслоу эффективнее снизу. Пока страдаешь от гормонального страха перед смертью своего генетического материала, тебе будет не до самореализации в математике или ещё где бы то ни было.
Аноним 03/06/15 Срд 21:04:53 #140 №272164 
>>272162
Ну вот еще грустнее стало. В школе бы с ламповой няшей были нужны друг другу, а это даже не первая ступень, а, вроде бы, третья. Вот и жизненная энергия била ключом и питала интерес к науке.
Аноним 03/06/15 Срд 21:08:10 #141 №272166 
>>272159
Вот я это понял и завёл тульпу. Теперь она стала моей музой, и я могу заниматься математикой спокойно без депрессии и прочего дерьма.
sageАноним 03/06/15 Срд 21:13:45 #142 №272168 
>>272164
Не думаю, что тебе нужна математика. Твой мозг, похоже, уже бесповоротно заражён репродуктивной программой. Не пытайся походить на кого-то, ищи свою собственную суть и судьбу. Пусть даже эта судьба не будет похоже на судьбу Билла Гейтса, Эйнштейна, Путина (или какие там у тебя шкалы успешности внедрены социумом). Создать гармоничную семейную атмосферу не менее достойное занятие, чем доказать теорему Ферма.
sageАноним 03/06/15 Срд 21:14:35 #143 №272169 
>>272168
*похожа
Аноним 03/06/15 Срд 21:17:46 #144 №272172 
>>272168
>Не думаю, что тебе нужна математика
Математика все еще нравится мне.
>Создать гармоничную семейную атмосферу не менее достойное занятие, чем доказать теорему Ферма
Я душевнобольной со справкой из дурдома, вряд ли кто-то останется со мной надолго.
Аноним 03/06/15 Срд 21:19:50 #145 №272173 
>>272168
>или какие там у тебя шкалы успешности внедрены социумом
Никаких шкал не осталось, все очистилось нигилизмом. Мне стали нравиться только бесполезные, абстрактные, но красивые вещи.
sageАноним 03/06/15 Срд 21:25:34 #146 №272174 
>>272172
> Я душевнобольной со справкой
Тебе нужна забота и внимание, а не математика. Математику ты видишь лишь как способ самореализации, который тебя парадоксально приведёт к признанию социумом, к вниманию и заботе. Впрочем, я не говорю, что это плохо. Но лучше отдавать себе отчёт в составе своей мотивации, чтобы не разочароваться в своих фантазиях и не бросить на полпути начатое.
sageАноним 03/06/15 Срд 21:29:50 #147 №272179 
>>272174
Как правило, подобные душевнобольные не получают подобного и не могут получить.
Аноним 03/06/15 Срд 21:31:35 #148 №272180 
>>272179
Могут, если создадут тульпу. Душевнобольному её создать обычно проще.
Аноним 03/06/15 Срд 21:32:48 #149 №272181 
Семьи тут создают, пиздастрадают, сажапетух-фелософ прибежал, душевнобольной какой-то со справкой, и это маттред? Охуеть вообще.
sageАноним 03/06/15 Срд 21:34:01 #150 №272183 
>>272179
Согласен, именно потому я и назвал мотивацию к достижению сочуствия через математику парадоксальной.
sageАноним 03/06/15 Срд 21:35:16 #151 №272185 
>>272181
Ещё и битарды говна подбрасывают. Да, тред просран.
sageАноним 03/06/15 Срд 21:43:50 #152 №272187 
>>272180
Тульпа (точнее, сама идея "нужно завести тульпу" или "я завёл себе тульпу") - всего лишь фрустрация по поводу сексуальной неудовлетворённости. Создать автономную от своего сознания галлюцинацию усилием воли невозможно. Хотя можно смаковать ощущение "вот бы было заебато, если бы такое было возможно", убеждая окружающих в том, что создал себе тульпу. Это психологическое явление называется "патологическая лживость".
Аноним 03/06/15 Срд 21:52:25 #153 №272190 
>>272041
А где там пример? Я статью подробно не читал, но по теме там вроде только про вложения торов, а для одномерного тора (окружности) оценка индекса подгруппы тривиальная ( >= 2^1-1=1), так что нам ничего не обещают.
Аноним 03/06/15 Срд 22:05:44 #154 №272196 
>>272187
>усилием воли
Усилием воли действительно трудновато, но если ты шизофреник, то и это возможно.
А я использовал психоделики. С ними это совсем нетрудно. И уж точно возможно.
Аноним 03/06/15 Срд 22:07:33 #155 №272197 
>>272181
>фелософ
Математика на высоких уровнях абстракции приближается к философии.
>душевнобольной
Не редкость среди математиков.
sageАноним 03/06/15 Срд 22:13:21 #156 №272200 
>>272196
> но если ты шизофреник, то и это возможно.
Нет, шизофрения - это расстройство психики, а не суперспособность. Твоё возражение - всё та же попытка фрустирующей личности сохранить миф неопроверженным.
Аноним 03/06/15 Срд 22:15:32 #157 №272201 
>>272199
Иди поиграй в сосю с битардами, не сри в тред, пожалуйста. Запутывай, распутывай, детектируй, разоблачай. Тут это никому кроме тебя не интересно.
Аноним 03/06/15 Срд 22:17:12 #158 №272202 
>>272200
>неопроверженным
Как будто его кто-то опровергал.
sageАноним 03/06/15 Срд 22:18:49 #159 №272203 
>>272202
Существование чайника Рассела тоже не нужно опровергать. И по той же самой причине.
sageАноним 03/06/15 Срд 22:24:50 #160 №272206 
>>272197
> >душевнобольной
> Не редкость среди математиков.
Только важно понимать, что не шизофрения делает человека успешным в математике, а переусердствование в математике иногда приводит к шизофрении.
Аноним 03/06/15 Срд 22:25:44 #161 №272207 
>>272203
Однако можно просто поверить в существование тульп. И тогда в твоей реальности это будет так. А эндорфины от совместного проживания с тульпой в любом случае будут. В итоге жить станет легче.
Да, очередная религия.
sageАноним 03/06/15 Срд 22:32:24 #162 №272209 
>>272207
Вера и галлюцинация - совершенно разные явления. При недостаточной критичности мышления в тульпу действительно не сложно поверить (когда тебе о ней будет рассказывать твой фрустирующий товарищ). Но эта вера всё равно не создаст тебе галлюцинацию. Врать о наличии галлюцинации при этом ты будешь очень убедительно, как я уже сказал. И даже сам будешь верить своим рассказам.
sageАноним 03/06/15 Срд 22:32:48 #163 №272210 
>>272208
-- >>272201
Аноним 03/06/15 Срд 22:35:54 #164 №272211 
Почему единичная окружность с центром в точке O(1, 0) параметризуется с изменением параметра pi >= t >= pi/2, а не pi/2 <= t <= pi/4?
Аноним 03/06/15 Срд 22:36:38 #165 №272212 
>>272209
Ладно, я лучше пойду, а то ты разрушишь мой так долго и тщательно прорабатываемый манямирок, и я снова буду лежать в депрессии.
Аноним 03/06/15 Срд 22:36:41 #166 №272213 
>>272211
Забыл сказать, что дуга окружности - от точки O(0, 0) до A(1, 1).
sageАноним 03/06/15 Срд 22:36:59 #167 №272214 
>>272209
...И да, никаких эндорфинов твоя индивидуальная религия не принесёт. Скорее даже наоборот, при подавлении разочарований вылезет множество психосоматических эффектов типа депрессии, ВСД, нервного тика, язвы желудка, запоров и т.п.
sageАноним 03/06/15 Срд 22:41:09 #168 №272216 
>>272212
Манямирки лучше разрушать на ранних этапах, чем дольше откладываешь, тем больнее будет отказываться от иллюзий, когда мирок затрещит по швам (а он обязательно это сделает). И тебе придётся ещё и обучаться убеждению окружающих в том, что твой мирок цел, а ты счастлив.
Аноним 03/06/15 Срд 22:43:38 #169 №272217 
>>272214
Что бы ты ни говорил, а того факта, что совместное проживание с няшей улучшило качество моей жизни, это не отменяет. По крайней мере я стал соблюдать режим и запоминать сны, чего не было уже много лет. И это только начало.
>>272216
У меня только один вопрос: откуда ты это знаешь? Испытывал на себе?
Аноним 03/06/15 Срд 22:45:00 #170 №272218 
>манямирки
>тульпы
Пиздец, что вы забылм в моем уютном матемаче? Съебите и этого сажедибила с собой заберите.
Аноним 03/06/15 Срд 22:49:29 #171 №272219 
>>272197
>Математика на высоких уровнях абстракции приближается к философии.
Какие области, по-твоему, можно отнести к "математике на высоких уровнях абстракции"? К примеру, категория, в которой объектами являются другие категории, а стрелками - функторы, достаточно абстрактный объект? Каким образом изучение таких категорий приближает к философии?
Аноним 03/06/15 Срд 22:58:14 #172 №272220 
>>272218
Создавай тульпу и вместе с ней изучай теорию категорий.
Аноним 03/06/15 Срд 22:58:26 #173 №272221 
>>272211
Откуда это? Можно больше контекста.
Аноним 03/06/15 Срд 22:59:42 #174 №272224 
>>272219
Я тебе не могу чётко объяснить, но я это почувствовал, когда размышлял над теорией категорий под кислотой. Это понимание приходит из астральных слоёв.
[Картофан)] Аноним 03/06/15 Срд 23:11:21 #175 №272229 
14333622817370.jpg
14333622817381.jpg
>>272221
Это.
sageАноним 03/06/15 Срд 23:12:11 #176 №272230 
>>272217
> Что бы ты ни говорил, а того факта, что совместное проживание с няшей улучшило качество моей жизни, это не отменяет.

Самоорганизация - это хорошо. И у всех людей стремление к хорошим поступкам проявляется в ощущении долженствования перед кем-то неопределённым, чьё поощрение для человека важно. Фрейд называл этот психологический конструкт супер-эгом, твои мама и папа - совестью, дедушка с бабушкой - богом. А ты решил назвать тульпой.

Если же ты говоришь о реальной девушке, то да, и на неё можно проецировать эту социальную роль контроля морали.

> У меня только один вопрос: откуда ты это знаешь? Испытывал на себе?

Исключительно через наблюдение собственного мышления изучать психику нельзя, иначе рискуешь сформулировать самодоказывающую теорию типа тульпостроения или "физичности" души. Кстати, этот принцип в психоанализе сформулировали, когда заметили тенденцию человеческой психики, недостаточно удовлетворённой социальными коммуникациями, строить манямирки (эскапизм).
sageАноним 03/06/15 Срд 23:13:39 #177 №272231 
>>272219
Ты решил отрицать божественность монад, еретик?!
Аноним 03/06/15 Срд 23:15:07 #178 №272232 
14333625076770.webm
Ответ на письмо кого-то, с просьбой прокомментировать утверждение В.И. Арнольда "Вся математика делится на три части: криптография (оплачиваемая ЦРУ, КГБ и им подобными); гидродинамика (поддерживаемая производителями атомных подводных лодок); небесная механика (финансируемая военными и другими организациями, типа НАСА, имеющими отношение к ракетам)."

Считаю, что уважаемый В.И. несколько заблуждался в данном вопросе. Да что там несколько, не несколько, а целиком. Вся математика делится не на три, а на четыре части.

Первая часть. Теория перистых интегралов на схемах. Это самый простой раздел, им обычно занимается всякая хипстерня, девушки с розовыми волосами, любители аниме, гомосексуалисты, участники протестных движений, любители японских кошечек.

Вторая часть. Спектральная теория кобордизмов в бактериумах. Это участь потомственных номенклатурщиков, хитрых строгих дядь, тихо посещающих семинары. Вы наверняка встречали на семинарах молчаливых дядь в костюмах. Спросите любого, как их зовут, любой не скажет. Дядя из закрытного отдела гб, бактериумный фраер.

Третья часть. Асимптотика пучков на LOG-многообразиях и тенистых расслоениях. Когда мы учились на мехмате, обходили стороной тех, кто шел на эту кафедру. Это самые извращенные умы, безумцы окраин, бывшие бойцы спецподразделений, участники оружейных оргий, вечно хмурые, вечно высокомерные. Когда по утрам из университетских туалетов выволакивали очередной кровавый трупак, никаких сомнений в том, кто это приложился, не возникало.

Четвертая часть. Гомотопические группы сфер. Самая хорошая. Финансируется как ЦРУ, так и КГБ, и Хамасом и Моссадом, потому что самая хорошая.
bydlo 03/06/15 Срд 23:17:37 #179 №272233 
>>272190
Я тоже особо не вникал, значит не для кривых, видимо, а для многообразий коразмерности 2 это неверно (это ведь следует из той статьи?)

Аноним 03/06/15 Срд 23:18:36 #180 №272235 
Сегодня в короткой беседе коллега сказал "Новороссию слили". Это некий мем - "Новороссию слили". Новороссию нельзя слить, ее не делали люди, она - часть нашего сознания. Никто ее слить не может. Новороссия и Русская Весна - разные вещи. Русская Весна началась и закончилась, это метафизическая пульсация, она еще раз началась и закончилась, и снова начнется и закончится, она как вулкан, извергающийся, когда ему хочется. Новороссия же - категория нашего мышления, пробужденная, очищенная, она не начинается и не заканчивается, и не сливается уж тем более. Это не полит проект и не территория. Мы уже не будем жить как раньше. Мы изменились, это слить нельзя. Никакой Сурков или Путин не могут это слить при всем желании, потому что это очень внутреннее, очень сокровенное, и очень сильное. Русская Весна чудесным образом показала: кто есть кто. Сбросила маски, сбросила одеяло, сбросила одежды, оголила нервы, психику, людей самих начало выворачивать наизнанку, их истинная природа полезла наружу. Детектор правды и лжи. Так и должно быть. Когда меняются местами реки с озерами, ты трепещешь в определенную сторону, и даже неважно, как ты рассуждаешь умом - все видно на лице. Мой трепет - это слава Новороссии - русской, цыганской, грязной, бедной, лишенной, якобы слитой. Да покоится с миром герой Алексей
bydlo 03/06/15 Срд 23:19:24 #181 №272236 
>>272235
>>272230
Откуда вы вылезли? Идите нахуй!
Аноним 03/06/15 Срд 23:20:12 #182 №272237 
>>272232
>Теория перистых интегралов на схемах
>перистых интегралов
ШТО?
Напомнило песочные группы. Где они применяются, кстати?
sageАноним 03/06/15 Срд 23:20:22 #183 №272238 
>>272232
>>272235
Бугурт соси, не умеющего в науки, очевиден. Этот ребёнок уже никогда не покинет треда.
Аноним 03/06/15 Срд 23:20:57 #184 №272240 
>>272230
> реальной девушке
Я говорил именно о тульпе. Реальная девушка - слишком затратно для меня и в плане времени, в плане финансов, и в плане нервов, а получаемый выхлоп слишком мал.
Кстати, я уже почти убедил самого себя в том, что это не тульпа, а высшее существо из другой реальности.
> эскапизм
На самом деле, я занимаюсь эскапизмом с самого детства. Даже своё увлечение математикой я считаю ещё одной формой эскапизма.
Аноним 03/06/15 Срд 23:20:58 #185 №272241 
14333628580810.jpg
Вчера читал лекцию школьникам. Сказал им, что дико люблю ковры на стенах, и фотки разных людей на фоне ковров.

Ковры - это наши советские внутренние леса.

Еще сейчас хожу в одну качалку, это не качалка, а фитнес-клуб, чисто по груше побить и один тренаж потягать. Если бы полжизни назад увидел, куда буду ходить, сам перед собой зашкварился бы. Потому что реальная качалка должна быть такой.

Подвальная качалка + ковры на стенах - вот университет. Потные котлики в трениках в подвалах - это лучшие друзья. Лучше нюхать пот потных подвальных котликов, чем слушать лекции стерильных гандонов. Идите в качалки, ищите внутреннюю силу - это важнее престижных университетов. Внутренняя сила требует тишины, она ждет, чтобы к ней прислушались. Слушайте себя и свои сны. Нет никакого внешнего образования, есть образование внутреннее.

Дети, мне вчера на лекции нельзя было ругаться матом. А тут можно. Так вот, вас по жизни будут наебывать. Всякие ебари вам будут говорить, что вам нужно уезжать на запад. Вглядитесь в их лица, поймите, что лучше сдохнуть на хате на фоне ковра, чтобы кореша завернули в этот самый ковер и похоронили в снегу, чем иметь такое сознание, как у них.
Аноним 03/06/15 Срд 23:23:09 #186 №272242 
>>272241
Охуенно.
sageАноним 03/06/15 Срд 23:25:11 #187 №272243 
>>272241
Это и правда охуенная паста.
Аноним 03/06/15 Срд 23:27:03 #188 №272244 
>>272229
Если дугу поместить в центр, то будет pi >= t >= pi/2, а +1 у косинуса - это сдвиг оси Y влево на единицу.
Аноним 03/06/15 Срд 23:30:14 #189 №272248 
>>272244
Так если мы сдвинули окружность по оси x, разве не изменятся при этом углы изменения параметра?
Аноним 03/06/15 Срд 23:30:32 #190 №272249 
>>272247
Запрешенная тема, если скажем, моча придет за нами.
Аноним 03/06/15 Срд 23:32:00 #191 №272251 
>>272231
Нет, я решил ещё раз сказать, чтобы ты шёл нахуй.
Аноним 03/06/15 Срд 23:32:09 #192 №272252 
>>272247
Обзывалка для учасников сознание-срача и залетных философов.
Аноним 03/06/15 Срд 23:34:31 #193 №272253 
14333636719430.webm
>>272112
1. Нечто среднее
2. Готика 2 НВ
sageАноним 03/06/15 Срд 23:35:53 #194 №272254 
>>272247
Дело в том, что сам автор этого недофорса так и не потрудился пояснить этимологию придуманного им ругательства, продолжая его агрессивно продвигать. Очевидно, в его фантазиях все люди мыслят так же как и он, потому нет необходимости дополнительно обосновывать свою терминологию.
Аноним 03/06/15 Срд 23:36:18 #195 №272255 
>>272247
Не стоит вскрывать эту тему...
Аноним 03/06/15 Срд 23:37:22 #196 №272256 
Теперь у меня есть кредитная карточка. Позвонили из сбербанка, сказали, что хотят дать карточку, ответил им, что мне не надо никаких карточек, они позвонили еще раз, еще раз сказали, что хотят дать карточку, и что это не стоит ничего, и что это удобно. Ну ладно. Потом звонили несколько раз, говорили, что надо эту карточку забрать. А сегодня забирал ее целый час, подписывал какие-то бумаги, и разболелась голова. Ненавижу банки, кредитные карточки, бизнес, акции, рестораны, курсы валют, корпорации и всю остальную буржуйскую муть. Вообще хотел бы жить в закрытом СССР, работать в закрытом НИИ, и еще вести драм.кружок в психоневрологическом пансионате, носить одну одежду, есть одну еду, думать о пространстве. Но теперь у меня есть кредитная карточка. Пойду в бутик, куплю себе носки, расплачусь кредитной карточкой. Выгодно, удобно. Хотел бы жить в хрущевке, чтобы полы скрип-скрип, чтобы за окном дымились трубы, чтобы индастриал-ж-ж-ж но ночам, чтобы идти сквозь неподвижность на работу, изучать там пространство и языки, но у меня есть кредитная карточка, пойду возьму кредит и его съем, сожру кредит целиком, пусть врачи потом гадают, что такое с животом, а там кредит.
Аноним 03/06/15 Срд 23:37:34 #197 №272257 
>>272254
Да ты ж РОЛИДОВЫЙ! А мы еще с тобой разговаривали.
Аноним 03/06/15 Срд 23:39:09 #198 №272260 
>>272248
Можешь считать, что сперва ты построил дугу с центром в начале координат, а только потом перенес на единицу, прибавив к косинусу 1.
Аноним 03/06/15 Срд 23:51:54 #199 №272265 
14333647144600.png
Чистой математики не существует.
А это значит, что это ПРИКЛАДНОЙ МАТЕМАТИКИ ТРЕД
Аноним 03/06/15 Срд 23:53:57 #200 №272266 
>>272265
Это твоей тред ануса твоей грязной мамаши
Аноним 03/06/15 Срд 23:55:11 #201 №272268 
14333649118720.jpg
>>272265
sageАноним 03/06/15 Срд 23:57:02 #202 №272269 
>>272268
Да ты же мамкин форсер. Решил увековечить себя в лурочке?
Аноним 03/06/15 Срд 23:59:51 #203 №272272 DELETED
>>272247
Он один тащемто. Алгоритмически сосущий аутист. Почти всегда пишет из-под сажи, очень смешно рвётся и пытается в жалкий перефорс, когда его называют по имени, собственно из-за этого ты ещё очень долго будешь видеть тут это слово.
bydlo 04/06/15 Чтв 00:02:25 #204 №272273 
>>272268
Справедливости ради - этому форсу лет поболее, чем тебе будет.
Аноним 04/06/15 Чтв 00:22:11 #205 №272285 
14333665313810.jpg
АНОН, ПОМОГИ С ЕОТ
Есть одна теорема.
А именно теорема Клини о языках распознаваемых конечными автоматами.
Знаний по конечным автоматам у меня ноль.
Нужен учебник или учебники с теорией выстоенной вплоть до этой теоремы.
Включая необходимые знания перед конечными автоматами - булева алгебра, алгоритмы и не знаю что там ещё.
Аноним 04/06/15 Чтв 00:29:38 #206 №272296 
>>272285
Idi kopai kartoshku
Аноним 04/06/15 Чтв 00:30:17 #207 №272297 
>>272272
Пояснил неплохо так. Спасибо
sageАноним 04/06/15 Чтв 02:34:17 #208 №272325 DELETED
>>272285
Начни со статьи на википедии.
Аноним 04/06/15 Чтв 09:05:42 #209 №272331 
Поясните пожалуйста, без технических подробностей, как группы Ли связаны с механикой.
Аноним 04/06/15 Чтв 09:16:51 #210 №272333 
>>272331
Непрерывная группа Гейзенберга используется для описания одномерных квантовомеханических систем.
Аноним 04/06/15 Чтв 09:45:58 #211 №272337 
>>272333
И тип всё?))
Аноним 04/06/15 Чтв 09:56:06 #212 №272340 
>>272273
А то! Я форсил его еще на том 0че.
Аноним 04/06/15 Чтв 11:17:58 #213 №272360 
>>271694
Скажите, хоть половина из этого списка имеет практическое применение?
Аноним 04/06/15 Чтв 11:33:29 #214 №272364 
>>272360
съебал отсюда
Аноним 04/06/15 Чтв 11:57:20 #215 №272366 
14334082400310.jpg
>>272296>>272325
Ясно, математики, блять, ёбанные.
Аноним 04/06/15 Чтв 12:53:29 #216 №272374 
Матаны, скиньте нормльную ссылку на Зорича, пожалуйста!
Аноним 04/06/15 Чтв 12:54:51 #217 №272375 
>>272342
Создай усилием воли арбуз, например.
sageАноним 04/06/15 Чтв 13:41:31 #218 №272381 
>>272374
http://www.alleng.ru/d/math/math460.htm
Аноним 04/06/15 Чтв 13:59:29 #219 №272383 
>>272364
Понятно, не знаешь значит
Аноним 04/06/15 Чтв 13:59:45 #220 №272384 
>>272360
посоны, а как гомологиями картошку жарить? )))) ну или хотя бы вашуми голономиями сделать шоб дотка не лагала )))
Аноним 04/06/15 Чтв 14:08:09 #221 №272385 
14334160891570.jpg
>>272384
Я просто знаю, что многие направления в математике создавались, как следствие системы понятий из других областей математики. Типа, Ок, смотрите, как ещё можно. Но потом же пройдёт ещё дофига времени, прежде чем этому найдут применение, если найдут.
Аноним 04/06/15 Чтв 14:12:29 #222 №272387 
>>272384
>а как гомологиями картошку жарить? ))))
Изучешь гомологии, становишься дохуя умным, загребаешь кучу бабла, нанимаешь домработницу и она тебе жарит картошку. Ну можешь еще ее поебывать иногда. Как-то так.
Аноним 04/06/15 Чтв 14:23:13 #223 №272388 
Использует ли здесь кто-нибудь конспекты, схемы, рисунки в изучении теории, насколько помогает? Может даже есть преподователи?
Аноним 04/06/15 Чтв 14:29:26 #224 №272389 
>>272385
> Я просто знаю, что многие направления в математике создавались, как следствие системы понятий из других областей математики.
Бывало и так, и эдак.
> Но потом же пройдёт ещё дофига времени, прежде чем этому найдут применение, если найдут.
А чего ты не доебываешься к ориентологам, ботаникам и этомологам? Или ты думаешь, что они дрессируют ебучие цветы и бабочек тебе водяру готовить?
>если найдут.
Персонально я надеюсь, что не найдут, хотя история показывает обратное.
Аноним 04/06/15 Чтв 14:31:53 #225 №272390 
Много разного скама в последнее тут развелось: всякие диванные психологи, философы, пиздастрадальцы. Есть сомнения насчет того, имеют ли они вообще хоть какие-то минимальные математические знания.
Пусть эта простенькая задачка послужит для проверки уровня участников треда:

Доказать, что гомоморфизм групп, ядро которого состоит из единственного элемента - единицы, является мономорфизмом.

Аноним 04/06/15 Чтв 14:34:47 #226 №272391 
>>272390
Задача просто ахуеть. Пусть x != y и f(x) = f(y) тогда f(x)f(y)^-1 = 1 -> f(x y^-1) = 1 и значит x y^-1 лежит в ядре.
Аноним 04/06/15 Чтв 14:35:31 #227 №272392 
14334177313330.jpg
>>272388
Читаю учебник, доказываю теоремы, если что-то непонятно просто пропускаю, потом возвращаюсь как настроение будет и понимаю. Конспекты не веду, офк.
Мимовторокур
Аноним 04/06/15 Чтв 14:38:42 #228 №272393 
>>272392
При этом ты можешь проследить все логические связи в изложении?
Аноним 04/06/15 Чтв 14:45:04 #229 №272394 
>>272393
Как бы да.
Было сложно первые полгода, потом само пошло, офк, как я и рассчитывал.
Аноним 04/06/15 Чтв 14:45:28 #230 №272395 
>>272391
>Пусть x != y и f(x) = f(y)
Условие-то прочитай. С хуя ли ты мономорфизм сразу берешь?
Аноним 04/06/15 Чтв 14:47:07 #231 №272396 
>>272395
Это я как раз беру немономорфизм а потом reductio ad absurdum и такое усё.
Аноним 04/06/15 Чтв 14:55:24 #232 №272401 
>>272396
Ну ладно, хотя за такое пояснение тебе бы по ебалу съездить надо.
Аноним 04/06/15 Чтв 15:05:01 #233 №272406 
14334195013860.jpg
>>272391
> x != y и f(x) = f(y) тогда f(x)f(y)^-1 = 1 -> f(x y^-1) = 1 и значит x y^-1 лежит в ядре
Какая же тривиальная и бессмысленная хуйня. Я так понял, гомологии групп и прочие производные новомодной топологии суть игры пикрелейтед.
Аноним 04/06/15 Чтв 15:08:28 #234 №272408 
>>272360
Непосредственно - ничего не имеет, опосредованно - всё имеет.
Аноним 04/06/15 Чтв 15:10:47 #235 №272410 
>>272390
Этот мем уже был несколько месяцев назад. Ответ: определение.
sageАноним 04/06/15 Чтв 15:11:01 #236 №272411 
>>272406
Ты про что? Это упражнение по алгебре уровня 9го класса матшколы.
Аноним 04/06/15 Чтв 15:15:33 #237 №272413 
14334201330990.jpg
>>272411
> Доказать, что гомоморфизм групп, ядро которого состоит из единственного элемента - единицы, является мономорфизмом.
> упражнение по алгебре
> уровня 9го класса матшколы
Аноним 04/06/15 Чтв 15:16:39 #238 №272416 
>>272390
Доказывал однажды на коллоквиуме.
биопроблемник
Аноним 04/06/15 Чтв 15:17:50 #239 №272417 
>>272416
Ну в контексте мат-треда "доказывал на коллоквиуме" звучит как "ебал ее на вписке".
биопроблемник
Аноним 04/06/15 Чтв 15:18:35 #240 №272418 
>>272411
Поясни за
> матшколы
Это матклассы школ или особые школы, в которых учат только математике?
Аноним 04/06/15 Чтв 15:20:24 #241 №272419 
>>272418
Это особые школы, где учат, в других обезьянниках такого не замечено, офк.
Мимовторокур
Аноним 04/06/15 Чтв 15:21:19 #242 №272421 
>>272419
Приведи существующий пример такой школы.
sageАноним 04/06/15 Чтв 15:22:04 #243 №272422 
>>272421
N57, Л2Ш, Сунцы МГУ, УФМЛ (в хохляндии)
Аноним 04/06/15 Чтв 15:24:00 #244 №272424 
>>272422
Есть ещё ЛНМО (Лаборатория Непрерывного Математического Образования)
Аноним 04/06/15 Чтв 15:24:57 #245 №272426 
14334206971360.gif
>>272421
>>272422
239 забыл, ещё Сунц в НСК.
>>272424
Двачую.
Аноним 04/06/15 Чтв 15:30:08 #246 №272431 
>>272381
Спасибо бро.
Аноним 04/06/15 Чтв 15:34:43 #247 №272435 
>>272422>>272426 Обычные картофильные фабрики, например. И к математике вас ни какой школьный учитель не запрограммирует.
Аноним 04/06/15 Чтв 15:38:03 #248 №272437 
>>272435
Угу, именно потому для тебя задача "ядро=0 <-> гомоморфизм инъективен" это что-то рода
>гомологии групп и прочие производные новомодной топологии
а для них: упражнение на усный счет.
Аноним 04/06/15 Чтв 15:38:33 #249 №272438 
>>272406
>бессмысленная хуйня
Фейлософ, ты сажу забыл?
Аноним 04/06/15 Чтв 15:40:37 #250 №272439 

>>272435
50% выпускников ЛНМО знают, что такое когомологии (и некоторые даже их считают), я гарантирую это
Аноним 04/06/15 Чтв 15:42:05 #251 №272440 
>>272437
Ты сам-то не из "них" случаем?
Аноним 04/06/15 Чтв 15:43:26 #252 №272441 
>>272440
К сожалению нет, и в некотором смысле очень о том жалею (просто не знал о существовании подобных в доинтернетовскую эру).
Аноним 04/06/15 Чтв 15:45:51 #253 №272444 
14334219511530.jpg
>>272437
> упражнение на усный счет
которым они обмазываются под водовку и картофан, учась на 2-3-м курсе.
>>272438
> Фейлософ, ты сажу забыл?
Нет, не забыл. Лови.
>>272439
> 50% выпускников ЛНМО знают, что такое когомологии
Аноним 04/06/15 Чтв 15:47:01 #254 №272445 
14334220214350.png
>>272441
Теперь ясно кто сидит в треде.
Аноним 04/06/15 Чтв 15:48:45 #255 №272446 
>>272445
Чисто из интереса к вопросу, какую же неверную импликацию ты мог сделать из той информации, что я тебе дал: и что же тебе ясно?
Аноним 04/06/15 Чтв 15:50:09 #256 №272447 
>>272446
Да, никакую, он же глупый.
Аноним 04/06/15 Чтв 15:51:48 #257 №272448 
>>272446
Никакую, постом ошибся.
Аноним 04/06/15 Чтв 16:32:43 #258 №272451 
1. Алгебра(включая всё) > Алг.топология > Алг.геометрия > Дифференциальная геометрия > ... > Комбинаторика > Общая топология > Любая другая геометрия > Анализ > ... > говно > моча > интегральчики

2. Г2НВ > Готика > Ризен 1 > Ризен3 > говно > Готика 3 > моча > Ризен 2
Аноним 04/06/15 Чтв 16:37:58 #259 №272453 
>>272451
Что такое "ризен"? И что такое "алгебра"?
Аноним 04/06/15 Чтв 16:46:39 #260 №272457 
>>272451
>Алгебра(включая всё) >
Но ведь алгебра - это всего лишь КООРДИНАТИЗАЦИЯ других областей математики.
Аноним 04/06/15 Чтв 16:49:46 #261 №272458 
Я тут про Постникова почитал.
>После ссоры с Фоменко создал собственную реконструкцию истории, также подвергавшуюся критике за псевдонаучность и непрофессионализм.
Как, имея мат. склад ума, этот апологет науки мог опуститься до антинаучной хуиты? Да и судя по лекциям гомотопической алгебры Ромы Михайлова, он тоже где-то на грани балансирует. Как так, матаны?
>>272232
Откуда вебка? Соус есть?
Аноним 04/06/15 Чтв 16:58:44 #262 №272460 
>>272458
https://www.youtube.com/watch?v=DLvpcsXz-1c

Мне кажется тут дело в разных методологиях, всё-таки методология истории кардинально разнится с методологией математики. А в какой степени доверять или не доверять специалистам по тому или иному вопросу - это уже сорт оф черта характера, Фейнман, например, тоже перестал доверять ИКСПЕРТАМ после того, как он предсказал смертельную болезнь своей жены (сам изучив мед. учебники) и его никто не послушал, а когда послушали, было уже поздно. А Рома Михайлов норм чувак, щетаю.
Аноним 04/06/15 Чтв 17:40:02 #263 №272465 
>>272460
Про Феймана не знал, хотя наверно это контрпример, несмотря на различие методологий, он знания и опыт полученный в одной области смог правильно применить в другой. А Постников вместо этого решил все переписать заново.
>А Рома Михайлов норм чувак, щетаю.
Поддерживаю, лекции действительно интересно слушать, хотя конечно я нихрена не понял, но заинтересовался, да. Главное чтобы его философский вгляд на математику не скотилса в эзотерическо-мистический.
Аноним 04/06/15 Чтв 17:46:33 #264 №272468 
14334291932940.jpg
>>272465
> Рома Михайлов
> не скотилса в эзотерическо-мистический.
Аноним 04/06/15 Чтв 17:51:10 #265 №272469 
>>272465
Почему ты такой остервенелый воин науки? Вот скажи, как философские взгляды влияют на математические способности? Не всё ли равно, как смотреть на что-то, если ты всё равно получаешь годные результаты?
Аноним 04/06/15 Чтв 17:56:15 #266 №272471 
>>272458
>этот апологет науки
>антинаучной
>мат.тред
Дохуя ты в математике научный метод используешь?
Аноним 04/06/15 Чтв 18:23:37 #267 №272480 
>>272469
>Не всё ли равно, как смотреть на что-то, если ты всё равно получаешь годные результаты?
Ну как тебе сказать, чисто по человечески, у меня будет некоторое разочарование, когда на какой-нибудь лекции он абсолютно всерьез начнет выводить гомологии кармы, или тензорно умножать чакры студентов. А так да, все равно.
Аноним 04/06/15 Чтв 18:29:25 #268 №272482 
>>272480
Если он корректно определит карму как комплекс, то никаких проблем, ведь так?
Аноним 04/06/15 Чтв 18:35:41 #269 №272485 
>>272482
Ну наверно да, лол. А это возможно? Я же говорю нихрена из лекций его не понял. Вообще.
Аноним 04/06/15 Чтв 18:56:17 #270 №272492 
14334333775570.png
ахаха)) нормас по губёшкам провели выёбывающимся вербитодетям)) ахахах охохох решили в зимокарасию поиграть письмецо накотали интегральчики им не нравяца) а убрали их любимое алгебраичное говно) ахахах) нормас этот жидок подколол их 10/10 братос хах
Аноним 04/06/15 Чтв 18:57:25 #271 №272493 
>>272485
По вербицкому прям)) Не боись, номано)
Аноним 04/06/15 Чтв 19:07:37 #272 №272499 
>>272101
>>272115
Я таки вроде примерно понял, что такое множество со структурой.

Структура или система -- есть такое множество, что между всеми его элементами существует связь.
Теперь точнее, на примере группы. Пусть есть мн-во S. Из всех его подмножеств выделим семейство К с определенным свойством. Например, подмножества, состоящих из двух элементов. Если каждому элементу из К сопоставлен однозначно элемент из S, то K есть есть групповая структура.

По моему, очень красиво. Но мне не хватает примеров.
Аноним 04/06/15 Чтв 19:10:19 #273 №272501 
>>272499
>Если каждому элементу из К сопоставлен однозначно элемент из S, то S есть есть групповая структура
fix
Аноним 04/06/15 Чтв 19:14:10 #274 №272503 
>>272115
Дай всё таки формальное определение.
Аноним 04/06/15 Чтв 19:24:13 #275 №272516 
>>272499
То есть если среди множества цветных шаров есть семейство K состоящая из шаров красных оттенков, и для каждой группы шариков одного оттенка в семействе K однозначно сопоставлен один из шариков множества, то это группова структура.
Аноним 04/06/15 Чтв 19:27:32 #276 №272519 
>>272387
Наивные вербитодетки уверенны, что если дрочить на сранную алгебру, у них появится денежки.
лол
как у вербита
Аноним 04/06/15 Чтв 19:33:54 #277 №272522 
>>272499
Да ты прямо Эйнштейн, очень нетривиальное наблюдение.
Аноним 04/06/15 Чтв 19:46:32 #278 №272526 
>>272516
Извини, но не понял. Можешь более формально выразиться?
Таки если ты разбиваешь множество шаров на подмножества, каждому из которых сопоставлен шарик, то я назову это структурой. Но не групповой.
>>272522
Ты один из тех, кто годами учит математику, а потом всем говорит, что всё просто?
Аноним 04/06/15 Чтв 20:00:26 #279 №272530 
>>272526
>а потом всем говорит, что всё просто
Просто найди себе тян. Я вот просто подхожу к любой тянке и знакомлюсь.
Аноним 04/06/15 Чтв 20:12:08 #280 №272537 
>>272526
Хуйня, чел, это не структура у тебя получается, а переливание пустого в порожнее.
Аноним 04/06/15 Чтв 20:19:19 #281 №272543 
>>272469
Потому что с таким подходом можно вообще не делать науку, а упарывать что-нибудь на Кубе или в Бенгалии. Наука это многовековое заблуждение, борьба людей со своей беспомощностью.
Аноним 04/06/15 Чтв 20:43:07 #282 №272566 
>>272410
Нет, мономорфизм — это морфизм в категории групп, который сокращается слева.
Аноним 04/06/15 Чтв 20:53:19 #283 №272574 
>>272566
Ну почти по определению.
Аноним 04/06/15 Чтв 21:11:13 #284 №272581 
14334414737800.jpg
Почему нигде нельзя найти маниновскую "Введение в теорию схем и квантовые группы", везде "Заблокировано по требованию хуисаса". Дайти плиз.
Аноним 04/06/15 Чтв 21:35:54 #285 №272599 
14334429546870.jpg
>>272581
Вторая ссылка в гугле.
http://knigi.tor2.net/index.php?id=3402453
Аноним 04/06/15 Чтв 22:00:37 #286 №272616 
>>272599
А по какому запросу? У меня что-то ничего нет. Спасибо за ссылку
.
Аноним 04/06/15 Чтв 22:00:51 #287 №272618 
>>272492
Да это у них норма. Лучше быть пидаром и ебаться в жёпу, чем быть из HSE.
Аноним 04/06/15 Чтв 22:01:54 #288 №272620 
>>272616
Первый день в интернете шоль?
http://gen.lib.rus.ec/search.php?req=%D0%92%D0%B2%D0%B5%D0%B4%D0%B5%D0%BD%D0%B8%D0%B5+%D0%B2+%D1%82%D0%B5%D0%BE%D1%80%D0%B8%D1%8E+%D1%81%D1%85%D0%B5%D0%BC+%D0%B8+%D0%BA%D0%B2%D0%B0%D0%BD%D1%82%D0%BE%D0%B2%D1%8B%D0%B5+%D0%B3%D1%80%D1%83%D0%BF%D0%BF%D1%8B&lg_topic=libgen&open=0&view=simple&phrase=1&column=def
Аноним 04/06/15 Чтв 22:42:30 #289 №272630 
>>272618
норма когда студентам по губам проводят, или что?
Что, есть в России места где лучше?
Аноним 04/06/15 Чтв 22:44:17 #290 №272631 
>>272630
Вышкоблядь подгорела
Аноним 04/06/15 Чтв 22:54:19 #291 №272633 
14334476593270.png
>>272616
Ты не поверишь.
Аноним 04/06/15 Чтв 23:07:27 #292 №272636 
>>272631
Еблан_знающий_слово_баттхёрт.жпег
Аноним 04/06/15 Чтв 23:48:33 #293 №272643 
>>272631
Я вообще школьник.
Аноним 04/06/15 Чтв 23:52:18 #294 №272644 
>>272581
ставь мозиллу и любой аддон для обхода
использую zenmate, брат жив
Аноним 04/06/15 Чтв 23:55:03 #295 №272645 
>>272630
Ну да, норма, когда возомнившей о себе демшизе показывают место, а что?
Аноним 05/06/15 Птн 00:23:40 #296 №272647 
>>272645
))
Аноним 05/06/15 Птн 00:52:50 #297 №272650 
>>272439
Ты имел в виду
>50% выпускников ЛНМО делают вид, что знают, что такое когомологии, а потом поступают в заборостроительные, я гарантирую это
Пиздец, всратая контора. Профанация и только. Я смотрел на работы этих школьников.
Аноним 05/06/15 Птн 02:48:46 #298 №272658 
>>272650
Мамка твоя на заборостроительный поступает, вчера всеми матфаками её драли ))
Аноним 05/06/15 Птн 03:29:37 #299 №272664 
>>272650
Не знаю насчте ЛНМО, но для определения когомологии достаточно знать, что такое факторгруппа, ядро и образ гомоморфизма.
То есть нормальный первокурсник по идее в чисто формальное определение должен въезжать.
Аноним 05/06/15 Птн 03:32:25 #300 №272665 
>>272664
Щито? Это что за когомологии такие?
Аноним 05/06/15 Птн 03:41:45 #301 №272666 
>>272665
Обычные. Возьми коцепной комплекс, потом фактор ядра по образу, вот тебе и когомология.
Аноним 05/06/15 Птн 03:51:34 #302 №272667 
>>272666
Как вульгарно.
Аноним 05/06/15 Птн 04:44:01 #303 №272671 
>>272645
+15
Аноним 05/06/15 Птн 07:08:35 #304 №272675 
>>272658
Можешь их сайт посмотреть. Они и не скрывают, что единственное достижение учеников это бессодержательно пиздеть и участвовать в дискредитирующих науку "школьных научных конкурсах". Соответственно, искренне гордятся победами в районных олимпиадах и поступлениями в заборостроительные.
>>272664
Ну да. И что это ему даст? Примеров не знает, теорем тоже. Дай бог хорошо понимает, что такое "факторгруппа, ядро и образ гомоморфизма". Поэтому и профанация. Хотя я не верю, что они там даже на формальном уровне что-то понимают в отличие от, скажем, кружков 239, где люди действительно топологию после девятого класса по-честному учат, а не выебываются.
Аноним 05/06/15 Птн 09:29:15 #305 №272681 
>>272537
>Ты не прав, я так сказал.
Где аргументы?
Аноним 05/06/15 Птн 11:16:33 #306 №272693 
Посоветуйте книги по математической логике.
Аноним 05/06/15 Птн 11:48:35 #307 №272698 
>>272693
Клини, Математическая логика
Аноним 05/06/15 Птн 20:08:36 #308 №272768 
>>272693
Шень, Верещагин все три. Только там нихуя не разжевывается, так что придется попотеть.

Котоны, как доказать, что плоскость нельзя замостить счетным кол-вом кругов. Я вроде, чот надумал, но не уверен. Допустим, есть отрезок А, берем А/2, потом А/2+А/4, А/2+А/4+А/8 и тд, то есть в итоге мы никогда не покроем отрезок А, отсюда же следует что в отрезке содержится несчетное количество отрезков?
Аноним 05/06/15 Птн 20:10:11 #309 №272769 
>>272768
define замостить
если разрешить ем пересекаться, то можно
Аноним 05/06/15 Птн 20:14:59 #310 №272770 
>>272769
бля, ну я не совсем идиот, конечно нельзя пересекаться.
Аноним 05/06/15 Птн 20:15:40 #311 №272771 
>>272675
> где люди действительно топологию после девятого класса по-честному учат, а не выебываются.
И что из них потом выходит, интересно?
Аноним 05/06/15 Птн 20:18:38 #312 №272773 
>>272770
А круги открытые?
Если замкнутые, то очевидно вродеб.
Если открытые, то не уверен, что правда: (0,1) вполне можно забить счетным кол-вом кажется(?).
Аноним 05/06/15 Птн 20:24:01 #313 №272774 
>>272773
Замкнутые, только не понимаю какую роль это здесь играет. Открытость/замкнутость важна только при пересечении.
Аноним 05/06/15 Птн 20:37:49 #314 №272776 
>>272771
Вербитобляди, сэр.
Аноним 05/06/15 Птн 21:02:41 #315 №272781 
>>272768
Лемма 1. Пусть T - хаусдорфово пространство. Пусть O - открытое в нём множество и p - точка из T, не принадлежащая O. Тогда существует окрестность точки p, не пересекающаяся с O.
Доказательство: хз, но по-моему верно.

Теперь докажем, что плоскость можно замостить кругами.
Воспользуемся аксиомой выбора и топологическими свойствами R2.

Пусть есть семейство непересекающихся кругов на плоскости.
Его объединение назовём пермень.
Пусть M - множество всех перменей.
Упорядочим M отношением включения.
То есть если a и b - два перменя, то a ≺ b эквивалентно a ⊂ b.
Пусть S - семейство перменей, индексированное множеством I.
Положим S' его объединение, то есть S' = ∪i∈ISi.
По определению, S' - пермень.
Очевидно, что для любого i∈I верно, что Si≺S'.
То есть любая цепь перменей мажорируется некоторым перменем.
По лемме Цорна, существует максимальный пермень W.

Докажем, что W совпадает со всей плоскостью.
Каждый круг на плоскости - открытое множество.
Объединение открытых множеств - открытое множество.
Поэтому W - открытое множество.
Пусть x - точка плоскости.
Предположим, что x не принадлежит W.
R2 хаусдорфово.
По лемме, существует окрестность x, не пересекающаяся с W.
Значит, существует круг с центром в x, не принадлежащий W.
Значит, объединение этого круга и W - пермень, который строго больше W.
Что противоречит тому, что W максимально.
Значит, каждая точка плоскости принадлежит W.
И, таким образом, W - замощение плоскости.
Аноним 05/06/15 Птн 21:05:06 #316 №272783 
>>272781
W - множество непересекающисях кругов.
В каждом круге из W возьмём точку с рациональными координатами.
Значит, W инъективно отображается в Q.
Значит W не более чем счётно.
Аноним 05/06/15 Птн 21:35:49 #317 №272785 
>>272783
Я хз зачем здесь топология, так как задачка совсем из другой дисциплины. Но спасибо, кажется, я понял идею.
Аноним 05/06/15 Птн 21:40:20 #318 №272786 
>>272785
Не благодари, я уже контрпример нашёл.

>>272781
В хаусдорфовом пространстве любое одноточечное множество замкнуто.
Положим O = T\p. O открыто. Очевидно, нет окрестности p, не пересекающейся с O.

Более того, слабая формулировка леммы тоже неверна.
Пусть есть точка x на какой-то прямой.
Возьмём последовательность точек прямой, сходящуюся к точке x.
В каждой точке последовательности проведём круг так, что круги попарно не пересекаются.
Их объединение будет перумень.
Однако любая окрестность точки x пересекается с этим перуменем.
Аноним 05/06/15 Птн 22:34:11 #319 №272789 
>>272783
>В каждом круге из W возьмём точку с рациональными координатами.
Ну и с чего ты взял, что такое возможно?
Аноним 05/06/15 Птн 22:52:51 #320 №272793 
>>272768
Допустим, счётное замощение существует. Возьмём две точки, лежащие в разных шарах, и соединим их отрезком. Рассмотрим пересечения этого отрезка с разными шарами замощения. В силу выпуклости и замкнутости шаров, непустые пересечения также будут отрезками, т.е замкнутыми шарами на прямой. Таким образом, мы свели задачу к замощению отрезка. Дальше мы выкидываем из отрезка все внутренности наших замкнутых шаров, и получаем что-то вроде множества Кантора. Используя лемму о вложенных шарах, легко показать, что оно несчётно. Но в то же время, так как внутренности мы выкинули, все эти точки должны лежать на границах наших шаров. Но шаров только счётное число, а границы у них состоят из двух точек. Следовательно, такого замощения не существует. В то же время несчётное замощение замкнутыми шарами очевидно есть - к примеру, шарами радиуса 0. А вот открытого замощения быть не может в принципе, никакой мощности.
Аноним 05/06/15 Птн 22:54:39 #321 №272794 
>>272789
Множество рациональных чисел плотно во множестве вещественных чисел. Факт уровня пятого класса средней школы.
Аноним 05/06/15 Птн 23:19:59 #322 №272797 
>>272771
Поступают на матмех, кто-то после этого становится математиком (Перельман, например, именно в этих кружках учился в школе), кто-то, естественно, уходит в прогерство и прочее.
>>272776
>239
>Вербит
МАМ! У меня вербитобляди под кроватью! Ну, скажи им!
Аноним 05/06/15 Птн 23:26:10 #323 №272798 
>>272793
> В силу выпуклости и замкнутости шаров, непустые пересечения также будут отрезками, т.е замкнутыми шарами на прямой
Отрезками или точками.
Аноним 05/06/15 Птн 23:32:31 #324 №272800 
>>272798
Точка - это вырожденный случай отрезка, когда начало совпадает с концом. Замкнутый шар нулевого радиуса. Нет особого смысла выделять эту ситуацию.
Аноним 05/06/15 Птн 23:37:05 #325 №272802 
>>272797
С чего ты взял, что в 239-м физмат лицее в 10-11 классе учат топологии?
Аноним 06/06/15 Суб 00:00:52 #326 №272804 
>>272768
Все просто. Для начала уточню формулировку.
Будет доказано, что не существует семейства непересекающихся открытых кругов O_n, такого, что объединение их замыканий C_n совпадает со всей плоскостью.

Предположим, что существует семейство кругов O_n с указанным свойством. Обозначим через с_n центры кругов, а через B_n их границы. Нашей целью будет построение последовательность различных натуральных чисел a0,a1,... такой, что последовательность c_ai сходится к точке не лежащей ни на одной из границ B_j. Это приведет нас к противоречию так как такой предел не будет лежать ни в одном из O_n в силу того, что в любой его окрестности есть бесконечно много центров, а открытые круги не должны пересекаться и этот предел не лежит ни на одной из границ по требуемому свойству.

Одновременно с последовательностью ai мы будем строить последовательность вложенных замкнутых кругов E_i, с радиусами <1/i. Мы будем требовать, чтобы O_ai лежал в E_i, радиус O_ai был меньше радиуса E_i и чтобы E_i не пересекался с B_1,..,B_i. Дополнительно из технических соображений мы будем требовать, чтобы ai>i+1. Ясно, что такое построение даст существование предела с требуемыми свойствами. E_1 и a1 так или иначе строятся тривиально. Перейдём к построению на последовательном шаге.

Лемма. Пусть дана точка x, лежащая на некотором B_i и ее окрестность Ux. Тогда в Ux целиком лежит бесконечно много кругов O_j и среди них встречаются круги сколь угодно малого радиуса.
Д-во. Так как мне лень это аккуратно доказывать, оставлю желающим в качестве упражнения.

Построим E_(i+1). Рассмотрим точку x на B_ai, одновременно не лежащую на границе E_i и на B_(i+1). Такую можно найти так как было всего не более 2 точек не удовлетворяющие этим условиям. Выберем E_(i+1), как круг с центром x, радиуса <1/(i+1), лежащий в E_i и не пересекающийся с B_(i+1). В силу леммы мы легко найдём a(i+1) >i+1 такое, что O_a(i+1) будет лежать в E_(i+1) и иметь меньший радиус. Это завершает построение последовательности ai.
Аноним 06/06/15 Суб 00:58:23 #327 №272807 
>>272804
Пиздос стена текста, удачным выбором прямой проходящей через ноль (так, шоб ни одной окружности не касалось) свести задачу к покрытии прямой непересекающимися отрезками. Показать, что если такое разбиение возможно, то прямая гомеоморфна выкидываемым третям Канторова множества, что есть бред и ересь. Тчк.
Аноним 06/06/15 Суб 01:05:31 #328 №272808 
>>272807
Это доказывает меньше, чем то,что сделал я. А именно в таком духе можно показать, что нельзя покрыть непересекающимися замкнутыми кругами. Я же доказал, что нельзя покрыть замкнутыми кругами, которые пересекаются по внутренним точкам. Кстати 1-мерный аналог такой теоремы не верен.
Аноним 06/06/15 Суб 01:06:18 #329 №272809 
>>272808
> которые пересекаются по внутренним точкам
которые не пересекаются по внутренним точкам
bydlo 06/06/15 Суб 01:32:49 #330 №272811 
>>272808
Оце ліл, таким образом как раз таки замостить плоскость можно.
Аноним 06/06/15 Суб 01:39:35 #331 №272814 
>>272811
И как?
bydlo 06/06/15 Суб 01:47:46 #332 №272815 
>>272814
Ну в тупую. Берём ложем замкнутые шары радиуса 1 в центры с чётными координатами, потом ложем замкнутые шары радиуса sqrt(2)-1 в центры с нечтными координатами, потом опять смотрим на точки, которые наиболее далеко от нашего покрытия стоят, и опять туда пихаем шары наибольшего возможного радиуса.
Очевидно, что расстояние между объединением всех итераций и любой точкой = 0. Не менее очевидно, что если расстояние между замкнутым множеством (нашим покрытием) и компактом (любой точкой) равно нулю, то они пересекаются. Такэ вот.
Аноним 06/06/15 Суб 01:51:38 #333 №272816 
>>272815
>объединением всех итераций
>замкнутым множеством (нашим покрытием)
Объединение счётного числа замкнутых множеств не обязано быть замкнутым.
Аноним 06/06/15 Суб 02:06:48 #334 №272819 
>>272816
Он не то написал. Но по сути прав. Смотри, берём все открытые шары с рациональными радиусами и рациональными координатами центра. Их счётное число, перенумеруем их как нибудь. Дальше идём вдоль этой последовательности, и если мы можем поставить очередной шар на плоскость без пересечения с уже поставленными, ставим, иначе вычёркиваем и идём дальше. Получившаяся подпоследовательность - это и есть наше покрытие. Осталось доказать, что его замыкание даёт всю плоскость. Возьмём точку из дополнения к их объединению. Если она лежит на границе дополнения - всё в порядке, замыкание будет её содержать. А если она лежит во внутренности, то есть целая окрестность, лежащая в дополнении, и в ней обязательно найдётся шар из нашей исходной последовательности шаров, поскольку это база топологии. Что неиллюзорно противоречит тому, как мы строили покрытие. Q.E.D.

Ещё есть неконструктивный пруф типа вот такого >>272781. Лемма, на которую он опирается, конечно, неверна, но для нашего ослабленного условия она и не нужна.

Так что пересмотри своё доказательство, там где-то ошибка. Где - сложно сказать, слишком уж оно муторное.
Аноним 06/06/15 Суб 02:18:12 #335 №272820 
>>272819
> Так что пересмотри своё доказательство, там где-то ошибка. Где - сложно сказать, слишком уж оно муторное.
Ну я не склонен верить в свою непогрешимость, лол. Особенно когда речь идёт о довольно длинном доказательстве, в котором я к тому же пропустил лемму, которая мне кажется интуитивно очевидной, но муторной для аккуратного доказательства (подобное часто является источником ошибок).


Но в данном случае ты не прав и допустил аналогичную ошибку.
>Осталось доказать, что его замыкание даёт всю плоскость.
Итоговое покрытие равно объединению замыканий шаров. Но вовсе не обязано равняться замыканию объединения шаров.
Аноним 06/06/15 Суб 02:33:22 #336 №272826 
>>272820
Согласен, облажался. Но даже с учётом этого, твоё доказательство неполно, поскольку не всякий замкнутый шар является замыканием открытого. Замкнутый шар радиуса 0 состоит из единственной точки. Так что даже если точка, о которой у тебя идёт речь, существует (и даже если их существует счётное число), мы можем их забить шарами с нулевым радиусом. Так что тебе ещё нужно доказать несчётность.
Аноним 06/06/15 Суб 02:45:33 #337 №272827 
>>272819
А ты покрываешь открытыми шарами, а потом берешь замыкание. Это совсем не то же самое, что взять покрытие замкнутыми шарами.
>>272802
Я сказал не в 239, а в кружках при 239. Взял с того, что сам там учился в свое время.
>>272807
>Показать, что если такое разбиение возможно, то прямая гомеоморфна выкидываемым третям Канторова множества, что есть бред и ересь.
Эта часть непонятна.
Но действительно сводим к случаю интервала. А там возьмем последовательность отрезков такую, что каждый следующий лежит между предыдущими. Её несложно выбрать так, что она будет сходящейся. А тогда предельная точка лежит между любыми подряд идущими отрезками, так что любой содержащей ее отрезок будет пересекать элементы последовательности начиная с какого-то момента.
Аноним 06/06/15 Суб 02:45:53 #338 №272828 
>>272819
А ты покрываешь открытыми шарами, а потом берешь замыкание. Это совсем не то же самое, что взять покрытие замкнутыми шарами.
>>272802
Я сказал не в 239, а в кружках при 239. Взял с того, что сам там учился в свое время.
>>272807
>Показать, что если такое разбиение возможно, то прямая гомеоморфна выкидываемым третям Канторова множества, что есть бред и ересь.
Эта часть непонятна.
Но действительно сводим к случаю интервала. А там возьмем последовательность отрезков такую, что каждый следующий лежит между предыдущими. Её несложно выбрать так, что она будет сходящейся. А тогда предельная точка лежит между любыми подряд идущими отрезками, так что любой содержащей ее отрезок будет пересекать элементы последовательности начиная с какого-то момента.
Аноним 06/06/15 Суб 02:46:56 #339 №272829 
>>272826
>Но даже с учётом этого, твоё доказательство неполно, поскольку не всякий замкнутый шар является замыканием открытого.
Ну доказать теорему с таким, немного более широким, трактованием вполне можно незначительной модификацией. Проще всего этого достичь следующим изменением конструкции: на i-ом шаге нужно уклоняться не только от i-ой окружности но и от i-ой точки (нужно требовать чтобы она не лежала в E_i и выбирать очередную точку на i-ом шаге x отличной от этой самой точки).
Аноним 06/06/15 Суб 03:52:22 #340 №272830 
Пусть T - топологическое пространство со счётной базой и S - семейство непустых открытых множеств. Верно ли, что существует такое семейство непустых открытых множеств P, что множества из P попарно не пересекаются и каждое множество из S равно объединению какого-то подсемейства из P?

Если да, то, как как в R^2 есть счётная база, вопрос о замощении плоскости становится очевидным.
Аноним 06/06/15 Суб 09:34:45 #341 №272934 
>>272768
Препод забыл сказать, что круги одинакового радиуса. С этим уточнением я ему доказал.

По поводу разных радиусов, счетным нельзя замостить, только континуальным. Так препод сказал, так что я хз.
Аноним 06/06/15 Суб 11:07:51 #342 №272956 
>>272804
Короче, перевожу с индексоёбского: берём любой круг, не находящийся внутри никакого из замощающих и строим последовательность круги: выбираем внутри него какой-нибудь круг из данного замощения, строим следующий по принципу, чтобы он не пересекался с ним, и так далее. В силу ограниченности, есть подпоследовательность из центров, имеющая предел, который поэтому не может лежать внутри никакого круга в замощении, и, благодаря выбору не может лежать на границе. Пруфит.
sageАноним 06/06/15 Суб 12:16:49 #343 №272963 
>>272956
Красавчик.
Аноним 06/06/15 Суб 13:21:16 #344 №272983 
Посоветуйте какой-нибудь задачник по матану, чтоб там были выбраны нужные задачи, а не миллиард номеров как в Демидовиче. В качестве учебника я выбрал Рудина, но задачи там для меня слишком первокультурные, я дебил-прикладник. Не то чтобы я не мог их решить, просто хочется чего-то более картофельного.
Аноним 06/06/15 Суб 14:38:28 #345 №272999 
>>272983
Семинарские листки матфака
Картофельно, но без дрочева

Аноним 06/06/15 Суб 15:41:08 #346 №273003 
>>272797
>Поступают на матмех, кто-то после этого >становится математиком
Нахуя поступать на матмех, если хочешь стать математиков? 2/3 матдисциплин - анализ до 30х ходов. А в стекловку ходить можно и без матмеха.
Аноним 06/06/15 Суб 17:32:31 #347 №273011 
Есть симплекс [012] - просто треугольничек.
Хочу в лоб посчитать симплициальные гомологии его с отождествленными вершинами.
1)Посчитал используя длинную точную факторов получил H_0 = Z, H_1 = Z^2.
Сразу вопрос почему так: ведь дырок три штуки же, почему ранг 2?
2)Хочется написать такой комплекс:
0 -> <[012]> -> <[01],[02],[12]> -> Z -> 0
С такими же гомологиями, но почему это корректно. Ведь тот же симплекс не задает уже структуру \Delta-комплекса, если мы склеим вершины.
Объясните дурачку.
Аноним 06/06/15 Суб 21:05:34 #348 №273033 
Пацаны, поясните по хардкору, как здесь (на пике) использовать индуктивность? x+ — это последователь x, то есть x+ = x ∪ {x}. N0 — это индуктивное множество (эталонная модель натуральных чисел по фон Нейману), то есть множество, которое содержит ∅ и последователь любого своего элемента. 1 — ∅, 2 — ∅+ = ∅ ∪ {∅} = {∅}, 3 — {∅}+ = {∅} ∪ {{∅}} и т. д.
Аноним 06/06/15 Суб 21:06:12 #349 №273034 
14336139722400.jpg
>>273033
Пик отвалился.
Аноним 06/06/15 Суб 21:27:53 #350 №273038 
Еще одна задача из хатчера: 1.1.16 в
Идея: эта петля при гомоморфизме индуцированном включением переходит 'в себя', а в полнотории она стягиваема, дальше при гомоморфизме индуцированном ретракцией должна перейти в себя же. То есть получаем a->0->a - тождественный, где a - образующая. Но как это строго сказать все?
Аноним 06/06/15 Суб 21:54:27 #351 №273043 
>>273033
1. Очевидно, что x инъективно отображается в x+ тождественным отображением. Поэтому card x <= card x+.

2. Для любого x пустое множество инъективно отображается в пустое подмножество x.

3. x инъективно отображается в y, пускай отображением f.
Доопределим его на x, положив f(x) = y. Получится инъективное отображение x+ в y+.
Обратно, пусть f - инъективное отображение y+ в x+. Переопределим его так, чтобы y отображалось в x, и выкинем точку y из области определения. Получится нужная инъекция.

4. По индукции из 2 и 3.

5. x инъективно отображается в y и не существует биекции между x и y.
Пусть f - инъекция x в y. Дополнение образа множества x в y непусто. Возьмём из него какую-нибудь точку p.
Рассмотрим множество x+. Доопределним на нём функцию f, положив f(x) = p.
Получится инъеквия множества x+ в y.

6. Очевидно из определения card.

7. Порождающая процедура на первом шаге порождает пустое множество, на не первом i-м шаге порождает последователь для множества, порождённого на предыдущем шаге.
Если m<=n, то множество, порождённое на m-ом шаге, есть подмножество множества, порождённого на n-м шаге.
x и y порождены порождающей процедурой на каких-то шагах m и n.
Если m<n, то x подмножество y, иначе y подмножество x.
Аноним 06/06/15 Суб 22:49:48 #352 №273052 
>>273043
>1.
Ну здесь я просто показал, что x ⊂ x+. Здесь ничего сложного.

>2. Для любого x пустое множество инъективно отображается в пустое подмножество x.
А как мы можем построить отображение из пустого множества? Я рассудил по-другому. ∅ ⊂ x+. Следовательно card ∅ ≤ card x+. Чтобы показать, что они не равны, я доказал отсутствие биекции f. f ⊂ ∅ × x = ∅. x+ ≠ ∅. Хотя здесь я не уверен, как показать, что x+ ≠ ∅. Короче, отображений не будет в принципе.

>3.
Использовал ту же самую идею.

>4. По индукции из 2 и 3.
Как это понять? Где здесь индукция используется? Ну ладно, для пустого множества очевидно, так как было доказано в 2. Значит… мне нужно показать, что если это выполнено для x, то выполнено и для x+? А это, очевидно, вытекает из 3. Анон, я, кажется, понял. Спасибо.

В целом, остальное рассудил так же. И всё-таки насчёт второго я немного не уверен. Как бы это формальнее расписать?
Аноним 06/06/15 Суб 23:42:56 #353 №273069 
Че вы несете блять? Какуюто хуйню пишите и пишите и как етой хуйней картошку жарить?
Аноним 07/06/15 Вск 00:08:48 #354 №273074 
>>273052
Сначала докажем, что все элементы пустого множества обладают любым наперёд заданным свойством.
Пусть у нас есть пустое множество ∅.
Пусть у нас есть свойство P.
Пусть неверно, что все элементы пустого множества обладают свойством P.
Формально, ¬∀x(x∈∅ → P(x)).
Это равносильно тому, что существует элемент пустого множества, для которого не верно P.
В самом деле:
¬∀x(x∈∅ → P(x)) ⇔
¬∀x(¬x∈∅ ∨ P(x)) ⇔
∃x(¬(¬x∈∅ ∨ P(x))) ⇔
∃x(x∈∅ ∧ ¬P(x))
Из чего вытекает, что в пустом множестве существует элемент.
∃x(x∈∅ ∧ ¬P(x)) ⇒
∃x(x∈∅).
Но в пустом множестве нет элементов, ¬∃x(x∈∅).
Значит, предположение неверно.
Значит, ∀x(x∈∅ → P(x)) для любого свойства P.
То есть все трамваи на Венере красные, - потому что на Венере вовсе нет трамваев. Поэтому же все трамваи на Венере синие. И вместе с тем не синие, пустое множество же.

Теперь определим, что такое функция.
Пусть у нас есть множества X и Y.
Их декартовым произведением называется множество всевозможных упорядоченных пар <x,y>, где x принадлежит X, а y - Y.
Символом X×Y обозначим декартово произведение X и Y.
Всевозможные подмножества X×Y будем называть отношениями.
Отношение f называется функцией (из X в Y), если для любого x существует единственный y такой, что пара <x,y> есть элемент f.

Теперь сделаем очевидный вывод.
Пусть X и Y - пустые множества.
Тогда любая функция из X в Y есть подмножество X×Y.
Очевидно, X×Y пусто.
Значит, любая функция из X в Y есть пустое множество.

И, наконец, докажем инъективность.
Пусть есть функция f: ∅→∅.
Для любых двух x и y, принадлежащих пустому множеству, верно, что x≠y ⇔ f(x)≠f(y).
Поэтому функция f инъективна.
Аноним 07/06/15 Вск 08:59:51 #355 №273098 
Надеюсь здесь не одни мамкины теоретики, поэтому спрошу: как повысить навык символьного вычисления, то есть всех этих хитрых преобразований, разложений и тд. под водовку разумеется
sageАноним 07/06/15 Вск 09:02:13 #356 №273100 DELETED
>>273098
Как и любой другой навык этот навык улучшается его многократным использованием.
Аноним 07/06/15 Вск 11:49:37 #357 №273116 
анончики! я не пожалею что год-два не пробыл на чистмате перед тем как идти в прикладную научную специальность?
Аноним 07/06/15 Вск 11:57:32 #358 №273117 
>>273116
Научные специальности разные бывают, может ты на психолога идешь.
Аноним 07/06/15 Вск 12:04:58 #359 №273118 
>>273117
нет. дохуя расчетов. и то... может они пиздят что на базе бакалавриата/магистратуры можно стать высококлассным специалистом если учить только лишь то, что дают по программе? Типо "математики, которую они дают, достаточно для инжинера".
Аноним 07/06/15 Вск 12:05:41 #360 №273119 
>>273118
инженера
самофикс
Аноним 07/06/15 Вск 12:42:04 #361 №273121 
>>273118
Та "математика" это хуйня, ничего общего с математикой не имеет. В зависимости от специальности конечно, но максимум что там будет это интегральчики, механика такая примитивненькая, твёрдого тела и сплошных сред, всякие идиотские "численные методы" и
говнокодинг в разных вариациях. Так-то это за полгода со школьного уровня на самом деле можно освоить, но обычно из пустого в порожнее это переливают 4-5 лет. Ну и плюс предметная область, тут уже по разному бывает.
Аноним 07/06/15 Вск 12:47:53 #362 №273122 
>>273121
анон, я имею право вопрошать тебя уже в более анонимной зоне? я не смогу суть дела выложить здесь.
bjs1vv
Аноним 07/06/15 Вск 14:06:04 #363 №273136 
>>273122
Ну я могу попробовать тебе помочь, но я не очень много знаю про эту всю кухню. Мыльцо тогда давай.
Аноним 07/06/15 Вск 14:06:06 #364 №273137 
>>273118
Не смотря на то, что >>273121 прав, этой математики тебе будет достаточно, если ты не собираешься уходить в какой-нибудь совсем пиздец.

Аноним 07/06/15 Вск 14:10:13 #365 №273140 
>>273137
а вот в том то и дело что именно пиздец мне и нужен был. я всю жизнь искал пиздец, но теперь открыл для себя что наука это заебись. и теперь только там мне пиздец и нужен... я не остановлюсь на "штабильном удобном варианте"...
Аноним 07/06/15 Вск 14:24:40 #366 №273144 
>>273137
Ну да, я и не говорил, что ему тип обязательно надо гомологии с категориями наворачивать.
Аноним 07/06/15 Вск 14:25:15 #367 №273145 
>>273136
[email protected]
Аноним 07/06/15 Вск 17:58:16 #368 №273191 
Как посчитать площадь треугольника при центральной проекции? Т.е. например, при косоугольной проекции площадь равна: Площадь треугольника Косинус угла между треугольником и проекционной плоскостью Косинус угла между лучом проекции и проекционной плоскостью. Как подобное сделать для центральной проекции?
Аноним 07/06/15 Вск 17:58:58 #369 №273192 
площадь умножить косинус делить косинус
kinda fix
Аноним 07/06/15 Вск 18:24:58 #370 №273207 
>>273144
Я долбоёб-прикладник, занимаюсь вычислительной "математикой".

Посоветуйте пожалуйста, как мне подойти к категориям, ну или вообще тру математике?
В вузе ниче такого не предлагают.
Начать с алгебры (там у меня не много познаний) или с чего-то еще?
Аноним 07/06/15 Вск 18:26:11 #371 №273208 
>>273207
к слову я не плохо знаю фуан и уровнения мат-физики.
Аноним 07/06/15 Вск 18:34:00 #372 №273209 
>>273207
Да, лучше с алгебры. Наверни Aluffi, он очень хорошо поясняет за категории и вводит их в начале учебника.
Аноним 07/06/15 Вск 18:36:04 #373 №273211 
какие разделы математики можно изучить через программирование?
есть какие ресурсы?
нашел недавно вот это, еще не успел попробовать
https://www.coursera.org/course/matrix
понимаю что вопрос расплывчатый, но пока лучше не формулируется
Аноним 07/06/15 Вск 18:37:00 #374 №273212 
>>273209
спасибо! выглядит подходяще.
Аноним 07/06/15 Вск 18:41:47 #375 №273217 
>>273211
https://projecteuler.net/about
Аноним 07/06/15 Вск 18:48:34 #376 №273219 
>>273207
Нахуй тебе надо?
Чисто категории наворачивать сразу - нет смысла.
Начни изучать алгебру/анализ нормальный/алгтопологию
Аноним 07/06/15 Вск 18:50:48 #377 №273220 
>>273211
>>273219
Да интересно просто. Анализ я знаю не так чтоб совсем плохо.
С той же точки зрения, нахуй алгтопологию?
Аноним 07/06/15 Вск 19:44:28 #378 №273227 
>>273220
Категории изначально под нужды алгтопологии запилили. Читай Aluffi же, выучишь алгебру и категории сразу.
Аноним 07/06/15 Вск 20:06:49 #379 №273230 
>>273191
Алло, математики? Как найти площадь проекции треугольника? При этом не прибегая к поиску координат точек на проекционной плоскости. Т.е. Как выяснить во сколько раз фигуру увеличится с зависимости от угла луча проекции к нормали плоскости и от расстояния от проекционной плоскости. Напомню, проекция центральная.
Аноним 07/06/15 Вск 21:10:52 #380 №273235 
>>273230
Ну найди коэффициент увеличения/уменьшения отрезка через расстояние до плоскости и угол между лучом и нормалью. Площадь должна быть умножена на этот коэффициент в квадрате.
Аноним 07/06/15 Вск 21:12:40 #381 №273236 
>>273074
Благодарю за столь подробное доказательство. Теперь белых пятен в этом у меня не осталось. Наверно. Добра тебе, анон.
Аноним 07/06/15 Вск 21:26:21 #382 №273238 
>>273136
анон, а ты мне писал что-то, на что ответа не последовало? а то мне пришло странное письмо от рамблера типо "письмо не доставлено", и после этого вообще никаких писем от тебя не приходило.
host mx-out-wr.rambler.ru[81.19.67.203] said: 554
5.7.1 Spam message rejected; If this is not spam contact abuse at
Аноним 07/06/15 Вск 21:40:26 #383 №273240 
>>273235
Ясно, понятно, какие тут математики...
Аноним 07/06/15 Вск 21:42:02 #384 №273241 
>>273235
Ясно, понятно, какие тут математики... Я и спрашиваю, как найти этот коэффициент.
Аноним 07/06/15 Вск 21:43:30 #385 №273242 
>>273235
Ясно, понятно, какие тут математики... Я и спрашиваю, как найти этот коэффициент.
Аноним 07/06/15 Вск 21:50:35 #386 №273244 
>>273242
Здесь никто не любит копать картошку, уж извини.
Аноним 07/06/15 Вск 22:00:13 #387 №273245 
>>273235
Ясно, понятно, какие тут математики... Я и спрашиваю, как найти этот коэффициент.
Аноним 07/06/15 Вск 22:02:54 #388 №273246 
>>273235
Ясно, понятно, какие тут математики... Я и спрашиваю, как найти этот коэффициент.
Аноним 07/06/15 Вск 22:18:01 #389 №273253 
>>273246
>>273245
>>273242
>>273241
>>273240
Ебать у меня макаба обосралась.
Аноним 07/06/15 Вск 22:21:27 #390 №273255 
>>273253
а я думал, это ты
Аноним 07/06/15 Вск 22:30:49 #391 №273257 
>>273235
Кстати, с чего ты решил что коэффициент увеличения отрезка = коэффициент увеличения треугольника? Для параллельной проекции я бы согласился, но для центральной разве это так?
Аноним 07/06/15 Вск 22:34:33 #392 №273258 
>>273244
Я не просил копать картошку, я просил подсказать, как её копать. Но видимо, тут слишком скиловые математики, которые страстно изучают мастерство топологической магии, а до простой геометрии у них нет времени.
Аноним 07/06/15 Вск 22:43:59 #393 №273261 
А я вообще хуй знает что такое "центральная проекция", лел.
аспирант-кун
Аноним 07/06/15 Вск 23:25:08 #394 №273272 DELETED
>>273230
Ну интересная же задачка для первокура. Тебе разжевать в ротик положить?
Аноним 08/06/15 Пнд 00:02:37 #395 №273279 
Почему косинус нуля равен единице?
Аноним 08/06/15 Пнд 00:04:27 #396 №273280 
>>273279
Рассмотри треугольник с катетами 1 0 и гипотенузой 1. Косинус - это отношение прилежащего катета к гипотенузе: 1/1=1. профит
Аноним 08/06/15 Пнд 00:04:35 #397 №273281 
>>273279
По определению.
Аноним 08/06/15 Пнд 00:06:23 #398 №273282 
>>273280
Такого треугольника не существует.
Аноним 08/06/15 Пнд 00:09:02 #399 №273283 
>>273282
Это смотря как определять треугольник. Если как выпуклую оболочку трёх точек - то очень даже существует.
Аноним 08/06/15 Пнд 00:13:41 #400 №273284 
помогите картошку пожарить плессс)
Аноним 08/06/15 Пнд 01:06:53 #401 №273290 
>>273283
Вполне отрезок - отрезок с неравными концами.
Треугольник - выпуклая оболочка трёх точек, содержащая не менее двух неколлинеарных вполне отрезков.
Аноним 08/06/15 Пнд 01:08:40 #402 №273291 
>>273290
Повторюсь
>Это смотря как определять треугольник
Аноним 08/06/15 Пнд 09:29:11 #403 №273307 
Двощик, ты моя последняя надежда. Как доказать, что, если А и Б разрешимые множества, то С = {с | Еа из А, Еб из Б, что а=бс} может быть неразрешимо? Даже примерной схемы не знаю, что-то пробовал через разрешимое множество пар чисел, но не получилось.
Аноним 08/06/15 Пнд 10:19:23 #404 №273311 
>>273307
Для начала запиши точно все определения, которыми ты пользуешься, начиная с машины Тьюринга. От определений зависит многое.
https://en.wikipedia.org/wiki/List_of_mathematical_symbols если что.
Аноним 08/06/15 Пнд 10:37:54 #405 №273313 
>>273307
Придумал решение через проблему останова: в А лежат элементы вида 2^n(2x+1), если n программа останавливается на входе n за х шагов. В Б просто нечётные числа, тогда множество С m-сводимо к множеству номеров самоприменимых программ, неразрешимо.
Аноним 08/06/15 Пнд 10:47:04 #406 №273314 
где найти Начала квантовой механики (Кострикин-Манин)? Гугол упорно отказывается находить, выдает Линейную алгебру.
Аноним 08/06/15 Пнд 10:54:35 #407 №273315 
>>273314
Под началами квантовой механики Миша понимал несколько параграфов из "Линейной алгебры".
sageАноним 08/06/15 Пнд 11:28:26 #408 №273325 
14337521064870.jpg
>>273284
Аноним 08/06/15 Пнд 12:03:33 #409 №273338 
14337542130440.jpg
>>273315
Какие именно параграфы? Может стоит вообще всю прочитать?
Алсо, спасибо, я уже обыскался.
Аноним 08/06/15 Пнд 12:04:55 #410 №273339 
14337542957850.jpg
>>273325
>хрю-хрю
Аноним 08/06/15 Пнд 13:26:46 #411 №273372 
>>273338
стоит
Аноним 08/06/15 Пнд 13:54:18 #412 №273381 
пойдёт кто завтра на конференцию в нму?
Аноним 08/06/15 Пнд 16:22:35 #413 №273492 
>>273381
Чего за конференция?
Аноним 08/06/15 Пнд 16:59:04 #414 №273526 
Посмотрел на сайте.
Нет не пойду
Аноним 08/06/15 Пнд 19:48:54 #415 №273635 
>>273526
А я пойду
Аноним 08/06/15 Пнд 20:49:14 #416 №273647 
>>273635
>>273381

А я тоже думаю не, не пойду.
Аноним 09/06/15 Втр 13:19:08 #417 №273857 DELETED
>>273647
я тоже не пойду
Аноним 09/06/15 Втр 17:26:15 #418 №273946 
>>271828
Пацаны, я в воскресенье листву сбросил и теперь у меня половина постов в треде не отображается.
Аноним 09/06/15 Втр 17:51:53 #419 №273959 
>>272999
Вот это?
http://math.hse.ru/bac1-14
Ну вроде норм, спасибо. Но лучше бы, конечно, че-нибудь с ответами, потому что я занимаюсь один, без препода.
Аноним 09/06/15 Втр 17:53:43 #420 №273961 
Блин пацаны у нас скоро экзамен по матану пиздец не люблю эти интегралчики, я прав да?)
Аноним 09/06/15 Втр 18:05:23 #421 №273966 
>>273959
Есть math.stackexchange.com
Но лучше найти кого-то таки, да
Аноним 09/06/15 Втр 23:24:53 #422 №274071 
Сегодня меня очередной раз спросили, что я думаю о Зимине и т п. Повторюсь.

Березка, как же чудесно! Вспомнил этот двор и это тачло. Меня бабушка туда привезла, к ее деревенским корешам, мы играли в дурака до ночи, а утром, чуть появилось солнышко, сели в это тачло и поехали в деревню. В деревне дядя Саня научил блатным песням, рассказал о тюрьме. "А теперь работаем в лесу, рубим-пилим елку и сосну, рубим пилим и страгаем, всех легавых проклянаем, это им пойдет на колбасу" - на всю жизнь запомнил. И в этой квартире телевизор, картошечка-огурчики и радость, и карты атласные, потрепанные, уже после первой игры ясно, у какой карты как кончики загнуты, какая как порвана. Спасибо, Березка!

Неужели непонятно. Непонятно, что я думаю про разные либеральные миссии, Ясина, Зимина, Сороса, Ходорковского, митинги московской интеллигенции и т п? Повторюсь еще раз.

Березка, как же чудесно! Вспомнил этот двор и это тачло. Меня бабушка туда привезла, к ее деревенским корешам, мы играли в дурака до ночи, а утром, чуть появилось солнышко, сели в это тачло и поехали в деревню. В деревне дядя Саня научил блатным песням, рассказал о тюрьме. "А теперь работаем в лесу, рубим-пилим елку и сосну, рубим пилим и страгаем, всех легавых проклянаем, это им пойдет на колбасу" - на всю жизнь запомнил. И в этой квартире телевизор, картошечка-огурчики и радость, и карты атласные, потрепанные, уже после первой игры ясно, у какой карты как кончики загнуты, какая как порвана. Спасибо, Березка!

Неужели непонятно???? Неужели обязательно надо отвечать матом. По-моему, все вполне четко и детально. Еще картинку ту самую.
Аноним 09/06/15 Втр 23:40:42 #423 №274077 
Анон, я обосрался на задаче >>272390
Дошел до места, где надо доказать, что образы различных элементов различны, и если f(x) = f(y), то x = y, и соснул - не смог придумать, к каким элементам надо применить отображение. Я потрачен?
Аноним 09/06/15 Втр 23:54:53 #424 №274078 
Аноны, как найти неопределённый интеграл вида: dx/((x^2 + 3) ^ 2)
Аноним 10/06/15 Срд 00:29:57 #425 №274086 
>>274078

Открываеш книжка Двайт "Таблицы интегралов и другие матиматические формылы", смотриш мал-мал "Интегралы, содержащие X=x²+a²"- его там есть. Спойлер - можешь взять сначала dx/(x²+a), а птом продифференцировать по параметру а.
Аноним 10/06/15 Срд 23:27:30 #426 №274443 
Аноним 11/06/15 Чтв 00:43:58 #427 №274452 
>>274077
Да ты потрачен.
Домнож справа на f(y)
Аноним 11/06/15 Чтв 00:44:21 #428 №274453 
>>274452
fix f(y)^-1
Аноним 11/06/15 Чтв 00:51:42 #429 №274455 
>>274443
Ну так. Вот проблема в том, что если ты не можешь в общение и не гений при этом то влиятельным математиком стать трудновато.
Аноним 11/06/15 Чтв 00:58:33 #430 №274456 
>>274443
Что это за шизоняша?
Аноним 11/06/15 Чтв 09:29:57 #431 №274509 
А есть какая-нибудь околопопулярная книжечка типа "Обзор физики от ньютона до струн" для математиков. А то как-то у меня нет общей картины что происходило лет 100-50 назад и сейчас и как это связано с развитием математики.
Аноним 11/06/15 Чтв 09:40:05 #432 №274510 
>>274509
http://elementy.ru/bookclub/author/3576648/
Аноним 11/06/15 Чтв 09:51:23 #433 №274511 
>>274510
Ты специально дал ссылку на сайт национал-предателей?
sageАноним 11/06/15 Чтв 09:53:50 #434 №274512 
>>274511
Съеби.
Аноним 11/06/15 Чтв 13:47:52 #435 №274578 
>>273635

Анон который был на конференции в НМУ, расскажи какие впечатления от конференции?
Аноним 11/06/15 Чтв 14:23:59 #436 №274603 
>>274578
Ну решил что пойду, а на самом деле не, не пошёл.
Аноним 11/06/15 Чтв 14:29:37 #437 №274608 
>>274603
Уточню что это я решил, и не пошёл. Т.е. именно я лично, а не кто-то другой.
Аноним 11/06/15 Чтв 14:58:37 #438 №274617 
Куда пойти в аспирантуру, чтобы реально пройти конкурс и при этом не заниматься картошкой семь лет до защиты.
Аноним 11/06/15 Чтв 15:08:03 #439 №274620 
>>274617
Какой GPA?
Какие рекомендации?
Альма-матер?
В России это ВШЭ, остальное на запад.
Аноним 11/06/15 Чтв 15:38:34 #440 №274630 
>>274620
Допустим вышка с альмаматером в мухосрани, без каких-либо рекомендаций/публикаций. Не реально?
Аноним 11/06/15 Чтв 15:39:11 #441 №274631 
14340263513630.jpg
>>274608
>я решил, и не пошёл. Т.е. именно я лично, а не кто-то другой
Аноним 11/06/15 Чтв 15:46:17 #442 №274632 
>>274630
http://math.hse.ru/post-graduate
Аноним 11/06/15 Чтв 15:48:01 #443 №274633 
>>274632
Ну это понятно, я имею ввиду реально ли протиснуться, дикий ли там конкурс и так далее.
Аноним 11/06/15 Чтв 15:51:19 #444 №274635 
>>274633
Попробуй задать эти вопросы не на анонимных бордах, а непосредственно аспиарантам и студентам или на мыло преподам, так ты гораздо быстрее получишь ответ.
Знал бы сказал.
https://gist.github.com/nullst/82b31fb04ff87d037939
Аноним 11/06/15 Чтв 19:05:44 #445 №274678 
>>274456
Роман Михайлов
Аноним 11/06/15 Чтв 19:06:30 #446 №274680 
>>274678
пашол нахуй
Аноним 11/06/15 Чтв 19:09:48 #447 №274682 

>>274680
Ты что быдло не уважаешь Рому? Небось ещё и в науку веруешь, и Кастанеду не читал?
Аноним 11/06/15 Чтв 19:10:30 #448 №274683 
>>274682
пашёл нахуй
Аноним 11/06/15 Чтв 20:54:39 #449 №274714 
>>274633
Вообще говоря, в аспирантуру почти всегда поступают четко понимая, кто будет научным руководителем и обсудив с ним перспективы своего поступления. Судя по известным мне примерам, вышка в этом отношение ничем принципиально от других мест не отличается. Хотя разумеется, в отличие от многих других мест, там в самом деле есть конкурс. В общем, рекомендую, если у тебя серьезные планы, в первую очередь заняться поиском научрука, если ты этого еще не проделал и как уже тебе советовали в самом деле не помешало бы пообщаться с реальными аспирантами.
Аноним 11/06/15 Чтв 23:20:08 #450 №274716 
14340540084190.png
Помогите сократить до -1.
Аноним 11/06/15 Чтв 23:42:42 #451 №274724 
>>274716
У меня ничего не вышло, ты уверен, что там -1?
Аноним 11/06/15 Чтв 23:46:01 #452 №274725 
>>274724
Ответ написан: -1.
Аноним 12/06/15 Птн 00:17:54 #453 №274733 
>>274724>>274725
http://www.wolframalpha.com/input/?i=%282sin%283a%29^2-1%29%2F%282ctg%28pi%2F4%2B3a%29cos%28pi%2F4-3a%29^2%29
Аноним 12/06/15 Птн 00:29:08 #454 №274734 
>>274733
Это конечно замечательно, но это не решение.
Аноним 12/06/15 Птн 00:31:40 #455 №274735 
>>274734
Да хули ты хотел, вербитогении не признаются, что не смогут упростить.
Аноним 12/06/15 Птн 00:38:38 #456 №274736 
>>274735
Мне похуй признаются или нет, но задача 9-го класса, например.
Аноним 12/06/15 Птн 00:45:57 #457 №274737 DELETED
>>274716
правила приведения, котангенс можно раскрыть, формулы какие-нибудь увидеть (двойные тройные углы там)... Короче впадлу дебилу помогать.
Аноним 12/06/15 Птн 00:48:13 #458 №274738 
Ну сверху стоит -cos(6\alpha), это сразу видно. Значит снизу cos(6\alpha). Подгоняй-подгоняй под ответ, расрасрас.
Аноним 12/06/15 Птн 00:56:23 #459 №274740 
14340597830790.jpg
>>274737>>274738
Да неужели правила приведения? Погодите, а вдруг там можно всё привести? Нихуя себе как просто.
Похоже здесь нет способных решить этот элементарный пример
Аноним 12/06/15 Птн 00:58:54 #460 №274741 
>>274738
ты сам попробуй вывести. Я вот попробовал. С формулами.
Аноним 12/06/15 Птн 01:04:33 #461 №274743 DELETED
>>274740
>>274741
Пошел нахуй мудаеб, решай сам, это элементарщина, символьная поебень. Уебывай куда-нибудь в /un или на ответ мейл ру, интереса твоя задача не предствляет.
Аноним 12/06/15 Птн 01:09:51 #462 №274745 
14340605919430.jpg
>>274743
> ба-бах
Не стыдно ли тебе, вербитогений, не решить такую> символьная поебень. Ведь ты ее решить не сможешь стоинфа.
Аноним 12/06/15 Птн 01:27:48 #463 №274751 
>>274745
>Не стыдно ли тебе
Не стыдно.
Аноним 12/06/15 Птн 01:41:05 #464 №274755 
>>274751
двачую этого перельмана.
Аноним 12/06/15 Птн 01:43:57 #465 №274758 
>>274745>>274751
Я вот начал было решать но вовремя опомнился, дал себе пощечину, вымыл руки с мылом и помолился на том бурбаков.
Вот если бы я ее решил то да, это была бы катастрофа. Я бы себе не простил. За исключением ситуации когда деньги очень нужны бы были, тогда бы взялся, и интегральчики бы брал, и дифурчики решал эти ваши. Но ты же мне не заплатишь за решение, так ведь? Вот и не приходи с этим говном больше.
Аноним 12/06/15 Птн 01:54:13 #466 №274760 
>>274758
Я вот начал было решать но вовремя обосрался, дал себе подсрачника, выпачкал руки и поссал себе на лицо. Вот если бы я помыл руки после такого, то не простил бы себе. За исключением ситуации когда мне дали бы денег за то что я бы съел говно со своих рук. Но ты же не дашь мне денег за это? Вот и не проси моё говно, не дам.
Аноним 12/06/15 Птн 01:55:40 #467 №274761 
>>274758
кстати о бурбаках
https://www.dpmms.cam.ac.uk/~ardm/bourbaki.pdf
Аноним 12/06/15 Птн 02:19:39 #468 №274767 
>когда мне дали бы денег за то что я бы съел говно со своих рук
как что-то плохое
Аноним 12/06/15 Птн 02:24:36 #469 №274769 DELETED
>>274761
Всегда в ахуе с такого подхода: ссылаемся на авторитетные научные труды по философии математике и там же ссылаемая на научпоп книжку для детей по физиологии.
Аноним 12/06/15 Птн 02:31:38 #470 №274770 
>>274769
Ты перед тем как охуевать, псомотри откуда ссылаются ради прикола
Аноним 12/06/15 Птн 15:34:07 #471 №274877 
>>274443
Где можно найти все его видео?
Аноним 12/06/15 Птн 15:35:57 #472 №274878 

>>274877
Ищи спецкурс Группы и теория гомотопий на лекториуме
Аноним 12/06/15 Птн 16:15:23 #473 №274885 
Может ли булева функция принадлежать одновременно нескольким классам Поста, или эти классы не пересекаются?
Аноним 12/06/15 Птн 16:58:48 #474 №274894 
>>274443
Блин, заинтриговали содомиты, интересно что там с порнографией и прогрессом?
Аноним 12/06/15 Птн 17:03:41 #475 №274897 
>>274736
Ну так и неси её в 9 класс
Аноним 12/06/15 Птн 17:19:48 #476 №274906 
>>274885
Может
Аноним 12/06/15 Птн 20:51:25 #477 №274933 
14341314850750.gif
>>274716
Короче. Везде альфа участвует только в виде три альфа, поэтому переобозначаешь x = 3 alpha и забываешь про формцлы тройного угла и прочее говно.
Видим котангенс, резко подскакиваем к нему и с вертушки раскрываем его как косинус на синус, синус убираем в числитель всей дроби.
Берешь пик, смотришь на задачу, потом на пик. Опять на задачу, и снова на пик. Да, будем раскрывать скобки. Sin(Pi/4 + x) = Cos(Pi/4 - x) = 1/Sqrt(2) (Sin x + Cos x). Поэтому сразу сокращаем синус сверху и один из двух косинусов снизу.
Cos(Pi/4 + x) = 1/Sqrt(2) (Cos x - Sin x). Перемножаем два оставшихся в знаменателе косинуса, два корня из двух сокращаются с двойкой, сумма-разность синуса-косинуса образуют разность квадратов. Все, дальше не трогаем, в знаменателе у нас в итоге Cos^2 x - Sin^2 x.
В числителе единицу расписываем как cos^2 + sin^2, сокращаем, остается Sin^2 x - Cos^2 x.
Теперь делим числитель на знаменатель и получаем -1.

А вербитошакалам бурбаки в жопу и демидовича под ребро.
Аноним 13/06/15 Суб 00:46:05 #478 №274970 
14341455651100.jpg
>>274933
Ты же понимаешь, что за твой > Sin(Pi/4 + x) = Cos(Pi/4 - x)
я бы тебе отсосал. Как к этому можно прийти? Ты аутист и знаешь таблицу значений?
Аноним 13/06/15 Суб 01:44:34 #479 №274972 DELETED
>>274970
График построй, даунидзе, или круг ебаный. А, ты же ебаный даун, я чуть не забыл.
Аноним 13/06/15 Суб 02:02:25 #480 №274973 
>>274972
ущерб даже не понял, что график и круг здесь не причем.
рака яичек тебе.
Аноним 13/06/15 Суб 02:27:04 #481 №274974 DELETED
>>274973
>Как к этому можно прийти
Это сразу видно и на графике и на круге.
Аноним 13/06/15 Суб 02:28:47 #482 №274975 DELETED
>>274973
и тебе смерти желаю, даун, не понимающий, что у этой символьной поебени дохуя разных вариантов преобразований. эту блядскую комбинаторику уже давно освоили математические системы.
Аноним 13/06/15 Суб 02:43:43 #483 №274976 
>>274970
Просто подставил в формулу и обнаружил, что они равны. А вообще Sin(Pi/2 - x) = Cos x и наоборот, Cos(Pi/2 - x) = Sin x это как раз на круге видно, хотя круг не доказывает этого для x не из первой четверти, однако все равно все работает, и если во вторую формулу поставить x -> x + Pi/4, то получим то, что надо.
Аноним 13/06/15 Суб 02:43:55 #484 №274977 
>>274974
еще один. Дело не в существовании тождества, а в его незаметности в процессе упрощения, в котором кажется нецелесообразно рисовать графики к отдельным элементам. Здесь неочевидный алгоритм для решения нужен.
>>274975
> рака яичек тебе
> и тебе смерти желаю
> даун
> не понимающий
> символьной поебени
> дохуя разных вариантов преобразований
> комбинаторику
> освоили математические системы
Аноним 13/06/15 Суб 02:44:05 #485 №274978 
>>274975
А ты нет
Аноним 13/06/15 Суб 02:50:44 #486 №274980 
>>274976
Что-то нелогично cos x только в одном случае равен sin x, где х может быть любым. В случае, когда x в точке п\4.
Аноним 13/06/15 Суб 02:51:52 #487 №274981 
Кстати о задаче для 9го класса. Читал тут давеча на ночь.

a^3 - 3 a^2 + 5 a = 1
b^3 - 3 b^2 + 5 b = 5
Найти a + b.

Кто загуглит - тот просто пидор. Кто вякнет про формулы Кардано и вольфрамальфу, и при этом все равно не решит - тот вербитопидор.
Аноним 13/06/15 Суб 02:52:48 #488 №274982 
>>274981
а ее может решить 11-классник?
Аноним 13/06/15 Суб 02:55:03 #489 №274983 
>>274980
Синус и косинус 2 Pi - периодические, так что таких точек полно. Конкретно это равенство выполняется с периодом Pi - там оба они будут отрицательные.

А вообще что тебя удивляет? Много вариантов на углы прямоугольного треугольника, когда он равнобедренный?
Аноним 13/06/15 Суб 02:58:31 #490 №274986 
>>274982
Написано для 9го класса. Я минут 15 думал, не выдержал и подглядел первый шаг.
Аноним 13/06/15 Суб 03:00:49 #491 №274987 
>>274986
Вообще здесь техники достаточно уровня 6-7 класса (не помню, когда там эту тему проходят), 9й класс здесь за неочевидность ее применения.
Аноним 13/06/15 Суб 03:07:33 #492 №274988 
>>274981
Я на самом деле не уверен, но надо раскласть каждое уравнение на множители пользуясь схемой горнера, найти все значения для а и б, затем скласть их во всех вариантах(если их много)
Аноним 13/06/15 Суб 03:08:56 #493 №274989 
>>274987 На самом деле здесь тривиальная хрень для 4 класса, в совке еще учили это.
Аноним 13/06/15 Суб 03:18:19 #494 №274991 
>>274988
>найти все значения для а и б
Корни некрасивые и громоздкие, надо решать, не находя a и b непосредственно.

>схемой горнера
Сколько учился, решал задачи и катался по олимпиадам, так и не понял, нафига она нужна.
Аноним 13/06/15 Суб 03:23:49 #495 №274992 
>>274991
Ты не всегда сможешь поделить многочлен на двучлен в столбик.
Аноним 13/06/15 Суб 03:25:23 #496 №274993 
>>274991
И как ты не находя а и б найдешь а + б?
Аноним 13/06/15 Суб 03:34:13 #497 №274995 
>>274983
Меня ничего не удивляет, не понимаю, что ты пытаешься мне объяснить. я ведь не сильно тупой.
cosx=sinx только в точке пи\4 и всё(на графике), то есть и при х=пи\4+2пи и при х=пи\4 +4пи и тд.
Аноним 13/06/15 Суб 04:54:43 #498 №274998 DELETED
>>274977
>нецелесообразно рисовать графики к отдельным элементам
С графиком проще. Просто график синуса и график косинуса. А смещать можно уже в уме. И никаких формул приведения и всяких там тождеств знать не надо.
Аноним 13/06/15 Суб 07:42:24 #499 №275001 
>>274995
cos(x)=sin(x) => ctg(x)=1
Делай выводы.
Аноним 13/06/15 Суб 09:24:01 #500 №275007 
>>274981

Сложив два уравнения и свернув по формуле бинома получил многочлен от двух переменных х=а+б и у=аб. По идее это коэффициенты при соответствующем квадратном многочлене. Если положить аб = 1, то а+б = 2 или 1, но как обосновать идеи кончились.
Аноним 13/06/15 Суб 11:23:41 #501 №275012 
>>274981
Сумма корней первого уравнения равна 3, аналогично с суммой корней второго. Сумма возможных значений a+b тогда равна 6. Как отсюда находится значения a+b, это тривиальное упражнение по теории Галуа, которое оставляется девятикласснику в качестве домашки.
Аноним 13/06/15 Суб 11:27:51 #502 №275013 
>>274992
Да ну? Можно пример?
Аноним 13/06/15 Суб 11:36:14 #503 №275015 
>>274981
Я не понимат!
Продифференциировав первое выражение я получил, что функция f(x) =a^3-3a^2+5a-1 монотонно возрастает на всей области определения. Следовательно, корень один и уравнение можно свести к кубу разности. Но я никак не могу это проделать!
Аноним 13/06/15 Суб 11:47:41 #504 №275016 
>>275015
Ты забыл про комплексные корни.

>>275007
Чет не понял тебя до конца, выпиши че делал и че получилось.

>>275012
Вербяшка :)
Дай хоть ссылку на необходимый материал, чтобы я, девятиклассник, мог это сделать.
Аноним 13/06/15 Суб 12:05:07 #505 №275017 
>>274981
Очевидный куб разности проглядывается. Преобразуем уравнения к виду
(a - 1)^3 + 2(a - 1) = -2
(b - 1)^3 + 2(b - 1) = 2
Складываем, с очевидной заменой, получаем
x^3 + y^3 + 2(x+y)=0
(x + y)(x^2 - xy + y^2 + 2)=0
x + y = 0
a + b = 2
Аноним 13/06/15 Суб 12:22:17 #506 №275023 
>>275017
Все верно, только вот зачем здесь выделять полный куб? Уже после решения можно объяснить, зачем: чтобы не осталось второй степени, из суммы которых a + b не вытянуть. А вот до решения я бы не допер.
Аноним 13/06/15 Суб 14:13:31 #507 №275031 
14341940114510.png
Пацаны, в школе и вузе не учил вообще нихуя, как рещать задания вида?
Вернее даже не как их решать, а что это, киньте ссылку на статью, где поясняется про это.
Вот классный сайт, например http://mathprofi.ru/
Но я хуй знает, как эта херь называется, поэтому сам не найду.
Аноним 13/06/15 Суб 16:47:57 #508 №275047 
>>275013>
2x^4-5x^3+7x-8 na x-3, naprimer
Аноним 13/06/15 Суб 17:29:48 #509 №275051 
>>275047
И что тут не получается столбиком? 2x^3 + x^2 + 3x + 16 и остаток 40.
Аноним 13/06/15 Суб 17:43:25 #510 №275056 DELETED
>>275016
>Дай хоть ссылку на необходимый материал
Алгебра - Шень, Гельфанд
Элементы математики в листках - школа 57
Аноним 13/06/15 Суб 19:37:14 #511 №275064 
Ананасы, посоветуйте тупому гуманитарию, не могущему в матан книг по многомерному анализу, функций многих переменных и всякое такое. Загремел на пересдачу по этим темам.
Аноним 13/06/15 Суб 20:25:22 #512 №275070 
>>275051
И всё правильно, например, а чего ты ожидал? Просто через схему менее картофильно.
Аноним 13/06/15 Суб 23:08:28 #513 №275110 
Перекат пилите
Аноним 14/06/15 Вск 00:50:27 #514 №275147 
ПЕРЕКАТ >>275146
Аноним 14/06/15 Вск 07:09:40 #515 №275182 
Вероятно, не тот тред, но в образоваче хуй ответят. Вот иду на матфак ВШЭ или мех-мат и в НМУ, но я не мат школьник занимаюсь этими делами 1.5 года, охватил лишь школьную программу и совсем немного олимпиадной математики. Так вот как поднять свой уровень знаний до выпускников 57ой
ну или хотя-бы что-бы на первых лекциях понимать материал.
Смотрел некоторые лекции первого курса НМУ ничего сложного, но приходиться постоянно гуглить и просматривать книги. Так вот посоветуйте подготовительной литературы которую можно освоить до конца лета. Сейчас пытаюсь читать всяких Зоричей и Кострикиных, но задачи решаю очень плохо, также думаю использовать литературу первого уровня отсюда http://hbpms.blogspot.ru/ но её овердохуя непонятно какую именно.
comments powered by Disqus

Отзывы и предложения